HESI RN: Maternity - Test Bank

Pataasin ang iyong marka sa homework at exams ngayon gamit ang Quizwiz!

A new mother, who is a lacto-ovo vegetarian, plans to breastfeed her infant. What information should the nurse provide prior to discharge.

a. Continue prenatal vitamins with B12 while breast feeding

What does a child in respiratory distress look like?

Apneic spells and grunting with prematurity or sepsis

The nurses assessing a 38-week gestation newborn infant immediately following a vaginal birth. Which assessment finding best indication that the infant is transitioning well to extrauterine life?

a. Cries vigorously when stimulated

The postpartum admission prescription for a client who delivered a healthy newborn includes one liter of lactated ringers with oxytocin 20units to infuse over 8 hours. How many milliunits /minute is the clients receiving?

0.4

Procardia (Nifedipine) what is it used for?

To stop preterm labor

Define hyper clonus

Hyperactive reflexes

What is Methergine for?

It helps reduce blood loss by contracting the uterus. It is an ergot alkaloid uterine stimulant. Do not give if mom has a high BP

PKU- prenatal history.

When both parents are carriers of an autosomal recessive gene, such as PKU, each child has a 25% change of being healthy.

When fetal movements or contractions compress the umbilical cord, variable decelerations can happen.

a. Maternal repositioning can alleviate cord compressions

The nurse is caring for a newborn who is 18inches long, weighs 4 pounds, 14 ounces, has a head circumference of 13 inches, and a chest circumference of 10inches. Based on these physical findings, assessment for which condition has the highest priority? a. Hyperthermia b. Hyperbilirubinemia c. Polycythemia d. Hypoglycemia

a. Hyperthermia

When teaching a gravid client how to perform kick (fetal movement) counts which instruction should the nurse includes.

a. If 10 kicks are not felt within 1hr, drink orange juice and count for another hour.

The parents of a newborn tell the nurse that their baby is already trying to walk. How should the nurse respond? A. Encourage the parents to report this to the healthcare provider B. Explain the newborns normal stepping reflex C. Acknowledge the parent's observation D. Schedule the newborn for further neurological testing

B. Explain the newborns normal stepping reflex

The nurse is performing a gestational age assessment on a full-term newborn during the first hour of transition using the Ballard (Dubowitz) scale. Based on this assessment, the nurse determines that the neonate has a maturity rating of 40-weeks. What findings should the nurse identify to determine if the neonate is small for gestational age (SGA)? (Select all that apply.)

- Frontal occipital circumference of 12.5 inches (31.25 cm) - Head to heel length of 17 inches (42.5 cm) - Admission weight of 4 pounds, 15 ounces (2244 grams) The normal full-term, appropriate for gestational age (AGA) newborn should fall between the measurement ranges of weight, 6-9 pounds (2700-4000 grams); length, 19-21 inches (48-53 cm); FOC, 13-14 inches (33-35 cm). This neonate's parameters plot below the 10% percentile, which indicate that the infant is SGA.

When explaining "postpartum blues" to a client who is 1 day postpartum, which symptoms should the nurse include in the teaching plan? (Select all that apply.)

- Mood swings - Tearfulness "Postpartum blues" is a common emotional response related to the rapid decrease in placental hormones after delivery and include mood swings, tearfulness, feeling low, emotional, and fatigued.

A healthcare provider informs the charge nurse of a labor and delivery unit that a client is coming to the unit with suspected abruptio placentae. What findings should the charge nurse expect the client to demonstrate? (Select all that apply.)

- dark red vaginal bleeding - increased uterine irritability - rigid abdomen.

A client whose labor is being augmented with an oxytocin (Pitocin) infusion requests an epidural for pain control. Findings of the last vaginal exam performed 1 hour ago, were 3 cm cervical dilation, 60% effacement, and a -2 station. What action should the nurse implement first? A. Determine current cervical dilation B. Request placement of the epidural C. Give bolus of intravenous fluids D. Decrease the oxytocin infusion rate

A. Determine current cervical dilation

During labor, the nurse determines that a full-term client is demonstrating late decelerations. In which sequence should the nurse implement these nursing actions? (Arrange in order.)

1 Reposition the client 2 Provide oxygen via face mask 3 Increase IV fluids 4 Call the healthcare provider To stabilize the fetus, intrauterine resuscitation is the first priority, and to enhance fetal blood supply, the laboring client should be repositioned (1) to displace the gravid uterus and improve fetal perfusion. Secondly, to optimize oxygenation of the circulatory blood volume, oxygen via face mask (2) should be applied to the mother. Next, the IV fluids should be increased (3) to expand the maternal circulating blood volume. Then, the primary healthcare provider should be notified (4) for additional interventions to resolve the fetal stress.

During labor, the nurse determines that a full-term client is demonstrating late decelerations. In which sequence should the nurse implement these nursing actions? (Arrange in order) a. Provide oxygen via face mask b. Reposition the client c. Increase IV fluid d. Call the healthcare provider

1. Reposition the Client - b. 2. Provide oxygen via face mask - a. 3. Increase IV fluid - c. 4. Call the healthcare provider - d. To stabilize the fetus, intrauterine resuscitation is the first priority, and to enhance the fetal blood supply, the laboring client should be repositioned (1) to displace the gravid uterus and improve fetal perfusion. Secondly, to optimize oxygenation of the circulatory blood volume, oxygen via face mask (2) should be applied to the mother. Next, the IV fluids should be increased (3) to expand the maternal circulating blood volume. Then, the primary healthcare provider should be notified (4) for additional interventions to resolve the fetal stress.

A 38-week primigravida client who is positive for group A beta streptococcus receives a prescription for cefazolin 2grams IV to be infused over 30mins. The medications available in 2 grams/100ml of normal saline. The nurse should program the infusion pump to deliver how many ml/hours?

1.6ml/hr

A new mother is afraid to touch her baby's head for fear of hurting the "large soft spot." Which explanation should the nurse give to this anxious client?

There's a strong, tough membrane there to protect the baby so you need not be afraid to wash or comb his/her hair.

How do we lower the levels if they are not severe?

This infant's bilirubin is beginning to climb, and the infant should be monitored to prevent further complications. Breast milk provides calories and enhances GI motility, which will assist the bowel in eliminating bilirubin.

A client at 32-weeks gestation is diagnosed with preeclampsia. Which assessment finding is most indicative of an impending convulsion?

Three plus deep tendon reflexes and hyper clonus are indicative of an impending convulsion and requires immediate attention.

A 38-week primigravida client who is positive for group A beta streptococcus receives a prescription for cefazolin 2grams IV to be infused over 30mins. The medications available in 2 grams/100ml of normal saline. The nurse should program the infusion pump to deliver how many ml/hours?

200 ml/hr.

A vaginally delivered infant of an HIV positive mother is admitted to the newborn nursery. What intervention should the nurse perform first?

To reduce direct contact with the human immuno-virus in blood and body fluids on the newborn's skin, a bath with an antimicrobial soap should be administered first.

A client at 32-weeks' gestation is diagnosed with preeclampsia. Which assessment finding is most indicative of an impending convulsion?

3+ deep tendon reflexes and hyper clonus

When assessing a client who is at 12-weeks gestation, the nurse recommends that she and her husband consider attending childbirth preparation classes. When is the best time for the couple to attend these classes?

30 weeks gestation at 30 weeks gestation is closest (of the options) to the time parents would be ready for such classes. Learning is facilitated by an interested pupil! The couple is most interested in childbirth toward the end of the pregnancy when they are psychologically ready for the termination of the pregnancy, and the birth of their child is an immediate concern.

A primigravida client who is 5 cm dilated, 90% effaced, and at 0 station is requesting an epidural for pain relief. Which assessment finding is most important for the nurse to report to the healthcare provider?

A platelet count of 67,000/mm3. Thrombocytopenia (low platelet count) should be reported to the healthcare provider because it places the client at risk for bleeding when an epidural is administered.

The nurse is preparing a client with a term pregnancy who is in active labor for an amniotomy. What equipment should the nurse have available at the client's bedside?

A single sterile glove, an amniotic hook, and Doppler to check fetal heart tones are the necessary equipment for performing an amniotomy.

When does the head return to its normal shape?

7-10 days

A client in active labor is admitted with preeclampsia. Which assessment finding is most significant in planning this client's care?

A 4+ reflex in a client with pregnancy-induced hypertension indicates hyperreflexia, which is an indication of an impending seizure.

The nurse is preparing to give an enema to a laboring client. Which client requires the most caution when carrying out this procedure?

A 40-week primigravida who is at 6 cm cervical dilatation and the presenting part is not engaged. When the presenting part is ballotable, it is floating out of the pelvis. In such a situation, the cord can descend before the fetus causing a prolapsed cord, which is an emergency situation.

What is the Silverman-Anderson Index?

A Silverman-Anderson Index has five categories with scores of 0, 1, or 2. The total score ranges from 0 to 10. A total score of 0 means the infant has no dyspnea, a total score of 10 indicates maximum respiratory distress.

Define Terbutaline sulfate (Brethine)

A beta-antagonist used for lung issues, but it also works (non-FDA approved) to slow preterm labor. Do not give if HR is high. a beta-sympathomimetic drug, stimulates beta-adrenergic receptors in the uterine muscle to stop contractions. The beta-adrenergic agonist properties of the drug may cause tachycardia, increased cardiac output, restlessness, headache, and a feeling of "nervousness".

A client at 32-weeks gestation is hospitalized with severe pregnancy-induced hypertension (PIH), and magnesium sulfate is prescribed to control the symptoms. Which assessment finding indicates the therapeutic drug level has been achieved?

A decreased in respiratory rate from 24 to 16 Magnesium sulfate, a CNS depressant, helps prevent seizures. A decreased respiratory rate indicates that the drug is effective. (Respiratory rate below 12 indicates toxic effects.)

A woman who thinks she could be pregnant calls her neighbor, a nurse, to ask when she could use a home pregnancy test to diagnose pregnancy. Which response is best?

A home pregnancy test can be used right after your first missed period. Home urine tests are based on the chemical detection of human chorionic gonadotrophin, which begins to increase 6 to 8 days after conception and is best detected at 2 weeks' gestation or immediately after the first missed period.

A client who suspects she is pregnant tells the nurse she has a peptic ulcer that is being treated with misoprostol (Cytotec), a synthetic prostaglandin C drug, how should the nurse respond? A) "You may be at risk for having a spontaneous miscarriage" B) "You may have an increased chance of having preeclampsia" C) "This medication will have no effect on your unborn child" D) "You may experience postpartum hemorrhaging after delivery"

A) "You may be at risk for having a spontaneous miscarriage"

The father of a 3-day old infant who is breast feeding calls the postpartum help line to report that his wife is acting strangely. She is irritable, cannot cope with the baby, and frequently cries for no appeared reason. What information is most important for the nurse to provide the father? A) Contact the clinic if the behaviors continue for more than two weeks or becomes worse B) Tell the father count the newborns number of soiled diapers over the next few days. C) A fluctuation in hormones in the early postpartum period can cause mood changes. D) Recommend giving supplemental bottle feedings to the baby between breast feeding.

A) Contact the clinic if the behaviors continue for more than two weeks or becomes worse

The home health nurse visits a client who delivered a full-term baby three days ago. The mother reports that the infant is waking up every 2 hours to bottle feed. The nurse notes white, curl-like patches on the newborns oral mucous membranes. What action should the nurse implement? A) Discuss the need for medication to treat curl-like oral patches B) Suggest switching the infant's formula C) Assess the baby's blood glucose level D) Remind mother not put the baby to bed with a propped bottle

A) Discuss the need for medication to treat curl-like oral patches

The nurse is caring for a newborn who is 18inches long, weighs 4 pounds, 14 ounces, has a head circumference of 13 inches, and a chest circumference of 10inches. Based on these physical findings, assessment for which condition has the highest priority? A) Hyperthermia B) Hyperbilirubinemia C) Polycythemia D) Hypoglycemia

A) Hyperthermia

A new mother who is breastfeeding her 4-week old infant and has type 1 diabetes, reports that her insulins needs have decreased since the birth of her child. What action should the nurse implement? A) Inform her that a decrease for insulin occurs while breastfeeding B) Advice the client to breastfeed more frequently C) Counsel her to increase her calories retake D) Schedule an appointment for the client with diabetic nurse educator

A) Inform her that a decrease for insulin occurs while breastfeeding

The parents of a male newborns have signed an informed consent for circumcision. which intervention should the nurse implement upon completion of the circumcision? A) Place petroleum gauze dressings on the site B) wrap the infant in warm receiving blankets C) Give a PRN dose of liquid acetaminophen D) Offer a pacifier dipped in glucose water

A) Place petroleum gauze dressings on the site

The nurse's assessment of a preterm infant reveals decreased muscle tone, signs of respiratory difficulty, irritability, and mottled, cool skin. Which intervention should the nurse implement first? A) Position radiant warmer over the crib B) Assess the infants blood glucose level C) Nipple feed 1 ounce 1% glucose in water D) Place the infant in side-lying position

A) Position radiant warmer over the crib

Which content should the nurse plan to include in a nutrition class for pregnant adolescents? (select all that apply) A) Take iron and calcium supplements daily B) Gain no more than 15 pounds during the pregnancy C) Increase food intake by 300 to 400 calorie /day D) Take folic acid supplement daily E) Maintain current protein intake

A) Take iron and calcium supplements daily C) Increase food intake by 300 to 400 calorie /day D) Take folic acid supplement daily E) Maintain current protein intake

A new mother asks the nurse about an area of swelling on her baby head near the posterior fontanel that lies across the suture lines. How should the nurse respond? A) That's called caput succedaneum. It will absorb and cause no problems. B) That is called a cephalohematoma. It will cause no problems. C) That is called a cephalohematoma. It can cause jaundice as it is. D) That is called caput succedaneum. It will have to be drained.

A) That's called caput succedaneum. It will absorb and cause no problems.

When performing daily head to toe assessment of a 1-day old newborn the nurse observes yellow tint to the skin on the forehead, sternum and abdomen. What action should the nurse take? A) measure bilirubin levels using transcutaneous bilirubinometer. B) review maternal medical records for blood type and Rh factor C) Prepare the newborn for phototherapy D) Evaluate cord a result

A) measure bilirubin levels using transcutaneous bilirubinometer.

Calculated by Nagele's rule, a primigravida client is at 28 weeks gestation. She is moderately obese and carrying twins and the nurse measures her fundal height at 27 cm. During the previous visit 3 weeks ago, the fundal height measured at 28 cm. Based on these findings, what should the nurse conclude? A. Fundal height measurement may indicate intrauterine growth retardation B. The healthcare provider needs to be notified immediately since this fundal height measurement is greater than expected C. Confirm the fundal height measurement with another nurse D. Recognize this as a reasonable fundal height measurement for this client

A. Fundal height measurement may indicate intrauterine growth retardation

The nurse if caring for a postpartum client who is complaining of severe pain and a feeling of pressure in her perineum. Her fundus if firm and she has a moderate lochia flow. On inspection, the nurse finds that a perineal hematoma is beginning to form. Which assessment finding should the nurse obtain first? A. Heart rate and blood pressure B. Abdominal contour and bowel sounds C. Urinary output and IV fluid intake D. Hemoglobin and Hematocrit

A. Heart rate and blood pressure

Following the vaginal delivery of a large for gestation age (LGA) infant, a woman is admitted to the ICU due to post-partum hemorrhaging. The client's medical record describes Jehovah's Witness notes as her religion. What action should the nurse take next? A. Inform the client of the critical need for a blood transfusion B. Obtain consent from the family to infuse packed red blood cells C. Clarify the clients wishes about receiving blood products D. Prepare to infuse multiple units of fresh frozen plasma

A. Inform the client of the critical need for a blood transfusion

A 30- year-old primigravida delivers a 9-pound infant vaginally after a 30- hour labor. What is the priority nursing action for this client? A. Observe for signs of uterine hemorrhage B. Encourage direct contact with the infant C. Assess the blood pressure for hypertension D. Gently massage fundus every four hours

A. Observe for signs of uterine hemorrhage

A diabetic client delivers a full term large for gestational age infant who is jittery. What action should the nurse take first? A. Obtain a blood glucose level B. Administer oxygen C. Feed the infant glucose water (10%) D. Decrease environmental stimuli

A. Obtain a blood glucose level

The nurse is caring for a postpartum patient who is exhibiting symptoms of spinal headaches 24 hours following delivery of a normal newborn. Prior to anesthesiologist's arrival on the unit, which action should the nurse perform? A. Place procedure equipment at bedside B. Apply an abdominal binder C. Cleanse the spinal injection site D. Insert an indwelling Foley catheter

A. Place procedure equipment at bedside

What is the normal bilirubin at 1 day old?

A. The normal total bilirubin level is 6 to 12 mg/dl after Day 1 of life.

A client at 20 weeks gestation comes to the antepartum clinic complaining of vaginal warts (human papillomavirus). What information should the nurse provide this client? A. Treatment options, while limited due to the pregnancy, are available B. The client should be treated with Penicillin G C. This client should be treat with acyclovir (Zovirax) D. Termination of the pregnancy should be considered

A. Treatment options, while limited due to the pregnancy, are available

A client at 33- weeks gestation is admitted with a moderate amount of vaginal bleeding and no contractions are noted on the external monitor. Which intervention should the nurse implement? A. Weight perineal pads B. Weight daily C. Measure intake and output D. Ambulate 15 minutes QID

A. Weight perineal pads

A multigravida client in labor is receiving oxytocin Pitocin 4mu/minute to help promote an effective contraction pattern. The available solution is Lactated Ringers 1,000 ml with Pitocin 20 units. The nurse should program the infusion pump to deliver how many ml/hr.?

ANS: 12

A 32- week primigravida who is in preterm labor receives a prescription for an infusion of D5W 500 ml with magnesium sulfate 20 grams at 1 gram/hour. How many ml/hours should the nurse program the infusion pump?

ANS: 25

Define amniotomy

AROM: may be performed by a midwife or obstetrician to induce or accelerate labor. The membranes may be ruptured using a specialized tool, such as an amnihook or amnicot, or they may be ruptured by the proceduralist's finger.

The nurse should explain to a 30-year-old gravid client that alpha fetoprotein testing is recommended for which purpose?

Assess for maternal pre-eclampsia Alpha-fetoprotein (AFP) is a screening test used in pregnancy. Elevated AFP may indicate an increased risk of neural tube defects such as anencephaly and spinal bifida.

A 42-week gestational client is receiving an intravenous infusion of oxytocin (Pitocin) to augment early labor. The nurse should discontinue the oxytocin infusion for which pattern of contractions?

Transition labor with contractions every 2 minutes, lasting 90 seconds each. Contraction pattern describes hyperstimulation and an inadequate resting time between contractions to allow for placental perfusion.

How many wet diapers per day?

At least 6-8

A woman who delivered a 9-pound baby boy by cesarean section under spinal anesthesia is recovering in the post anesthesia care unit. Her fundus is firm, at the umbilicus, and a continues trickles of bright red blood with no clots from the vagina in observed by the nurse. Which actions should the nurse implemented. A) Massage the fundus B) Assess her blood pressure C) Apply ice pack to perineum D) Let the infant breast feed

B) Assess her blood pressure

The nurses assessing a 38-week gestation newborn infant immediately following a vaginal birth. Which assessment finding best indication that the infant is transitioning well to extrauterine life? A) Heart rate 220 beats/minute B) Cries vigorously when stimulated C) A positive Babinski reflex D) Flexion of all four extremities

B) Cries vigorously when stimulated

A 26-week gestational primigravida who is carrying twins is seen in the clinic today. Her fundal height in measured at 29cm. Based on these findings what actions the nurse implement. A) Notify the healthcare provider of the finding B) Document the finding in the medical record C) Schedule the client for a biophysical profile D) Request another nurse measure the fundus

B) Document the finding in the medical record

The nurse is planning a class for pregnant women in the first trimester of pregnancy. Which information is most imported for the nurse to include in the class? A) Plan rest periods and increase sleep time to an hour per day when fatigue B) If any vaginal bleeding occurs, notify the healthcare provider immediately C) Since eating often relieves nausea, carry low fat snacks to eat whenever nausea occurs D) If morning dizziness occurs, rise slowly and sit on the side of the bed for one minute

B) If any vaginal bleeding occurs, notify the healthcare provider immediately

Which action should the nurse take if an infant, who was born yesterday weighing 7.5lbs (3,317grams) weights 7 lbs. (3,175grams) today. A) Monitor the stool and urine output of the neonate for the last 24 hours B) Inform and assure the mother that this is a normal weight loss C) Encourages the mother to increase frequency of breastfeeding. D) After verifying the accuracy of the weight, notify the healthcare provider.

B) Inform and assure the mother that this is a normal weight loss

A client who is anovulatory and has hyperprolactinemia is being treated for infertility with metformin, menotropins (Repronex, MENOPUR®), and human chorionic gonadotropin(hCG). Which side effects should the nurse tell the client to report immediately? A) Episodes of headache and irritability B) Nausea and vomiting C) Rapid increase in abdominal girth D) Persistent daytime fatigue

B) Nausea and vomiting

A newborn infant is receiving immunization prior discharge. Which action should the nurse implement? A) Give the first dose of the vaccine for rotavirus if any have diarrhea now. B) Obtain signed consent from the mother for administration of hepatitis B vaccine C) Prepare the first dose for DTaP D) Ask the mother if she wants the infant immunized

B) Obtain signed consent from the mother for administration of hepatitis B vaccine

A 39-week gestational multigravida is admitted to labor and delivery spontaneous rupture of membranes and contraction occurring 2 to 3 minutes. A vaginal exam indicates that the cervix is dilated 6cm, 90% effaced and the fetus is at a +2 station. During the last 45 minutes the fetal heart rate has ranged between 170 and 180 beats/minute. What action should the nurse implement? A) Obtain a blood specimen for hemoglobin B) Take an oral maternal temperature C) Straight Catheterize client D) Send amniotic fluid for analysis

B) Take an oral maternal temperature

The healthcare provides prescribes 10units/L of oxytocin (Pitocin) via IV drips to augment a client labor because she is experiencing a prolonged active phase. Which finding would cause the nurse to immediately discontinue the oxytocin? A) uterus soft B) contraction duration of 100 seconds C) four contractions in 10 minutes D) Early deceleration of fetal heart rate

B) contraction duration of 100 seconds

The nurse is teaching a client with gestational diabetes about nutrition and insulin need for pregnancy. Which content should the nurse include in this client teaching plan? A) Insulin production is decreased during pregnancy B) increase daily caloric intake is needed C) injection requirements remain the same D) Blood sugars need less monitoring in the first trimester

B) increase daily caloric intake is needed

A new mother, who is lacto-ovo vegetarian, plans to breastfeed her infant. What information should the nurse provide prior to discharge? A. Avoid using lanolin-based nipple cream or ointment B. Continue prenatal vitamins with B12 while breast feeding C. Offer iron- fortified supplemental formula daily D. Weigh the baby weekly to evaluate the newborns growth

B. Continue prenatal vitamins with B12 while breast feeding

The nurse is assessing a 38- week gestation newborn infant immediately following a vaginal birth. Which assessment finding best indicates that the infant is transitioning well to extra-uterine life? A. Flexion of all four extremities B. Cries vigorously when stimulated C. Heart rate of 22 beats/minute D. A positive Babinski reflexes

B. Cries vigorously when stimulated

A 3-hour old male infant's hands are feet are cyanotic, and he has an axillary temperature of 96.5 F, a respiratory rate of 40 breaths/min, and a heart rate of 165 beats/min. Which nursing intervention is best for the nurse to implement? A. Perform a heel- stick to monitor blood glucose level B. Gradually warm the infant under a radiant heat source C. Administer oxygen by mask at 2L/minute D. Notify the pediatrician of the infant's unstable vital signs

B. Gradually warm the infant under a radiant heat source

A new infant is receiving positive pressure ventilation after delivery. Based on which assessment finding should the nurse initiate chest compressions? A. Apgar score 7 B. Heart rate 54 C. Limp muscle tone D. Central cyanosis

B. Heart rate 54

A term multigravida, who is receiving oxytocin (Pitocin) for labor augmentation, is requesting pain medication. Review of the client's record indicates that she was medicated 30 minutes ago with butorphanol (Stadol) 2 mg and promethazine (Phenergan) 25 mg IV push. Vaginal examination reveals that the clients cervical dilation is 3 cm, 70% effaced, and at a 0 station. What action should the nurse implement? A. Medicate the client with an additional 1 mg of Stadol IV push B. Instruct the client to use deep breathing during a contraction C. Discontinue the Pitocin infusion D. Notify the healthcare provider

B. Instruct the client to use deep breathing during a contraction

Which topic is most important for the nurse to include in a nutrition teaching program for pregnant teenagers? A. Gestational diabetes B. Iron-deficiency anemia C. Excessive weight gain D. Elevated cholesterol

B. Iron-deficiency anemia

A client delivers a viable infant but begins to have excessive uncontrolled vaginal bleeding after the IV Pitocin is infused. When notifying the hcp of the client's condition, what information is most important for the nurse to provide? A. Total amount of Pitocin infused B. Maternal Blood pressure C. Maternal Apical Pulse rate D. Time Pitocin infusion completed

B. Maternal Blood pressure

A woman who delivered a normal newborn 24 hours ago complains, "I seem to be urinating every hour or so. Is that ok?". Which action should the nurse implement? A. Catheterize the client for residual urine volume B. Measure the next voiding, then palpate the client's bladder C. Evaluate for normal involution, then massage the fundus D. Obtain a specimen for urine culture and sensitivity

B. Measure the next voiding, then palpate the client's bladder

After breast-feeding 10 minutes at each breast, a new mother calls the nurse to the postpartum room to help change the newborns diaper. As the mother begins the diaper change, the newborn spits up the breast milk. What action should the nurse implement first? A. Wipe away the spit-up and assist the mother with the diaper change B. Turn the newborn to the side and bulb suction the mouth and nares C. Sit the newborn up and burp by rubbing or patting the upper back D. Place the newborn in a position with the head lower than the feet

B. Turn the newborn to the side and bulb suction the mouth and nares

The nurse is teaching a woman how to use her basal body temperature (BBT) pattern as a tool to assist her in conceiving a child. Which temperature pattern indicates the occurrence of ovulation, and therefore, the best time for intercourse to ensure conception?

Between the time the temperature falls and rises. In most women, the BBT drops slightly 24 to 36 hours before ovulation and rises 24 to 72 hours after ovulation, when the corpus luteum of the ruptured ovary produces progesterone. Therefore, intercourse between the time of the temperature fall and rise is the best time for conception. The human ovum can be fertilized 16 to 24 hours after ovulation.

A multigravida client at 41-weeks gestation presents in the labor and delivery unit after a non-stress test indicated that the fetus is experiencing some difficulties in utero. Which diagnostic test should the nurse prepare the client for additional information about fetal status?

Biophysical profile (BPP) BPP provides data regarding fetal risk surveillance by examining 5 areas: fetal breathing movements, fetal movements, amniotic fluid volume, and fetal tone and heart rate. The client's gestation has progressed past the estimated date of confinement, so the major concern is fetal well-being related to an aging placenta, not screening for fetal anomalies.

A client at 38- weeks gestation complaints of severe abdominal pain. Upon the nurse notes that the abdomen is rigid. A) Placenta previa B) Oligohydramnios C) Abruptio placenta D) Chorioamnionitis

C) Abruptio placenta

When planning care for a laboring client, the nurse identifies the need to withhold solids food while the client is in labored. What is the most important reasons for this nursing intervention? A) Nausea occurs from analgesics used during labor B) Autonomic nervous system stimulation during labor decrease peristalsis C) An increased risk of aspiration can occur if general anesthesia is needed D) Gastric emptying time decreases during labor

C) An increased risk of aspiration can occur if general anesthesia is needed

Following the vaginal delivery of a large-for-gestation-age (LGA) infant a woman is admitted to the intensive care unit due to postpartum hemorrhaging. The client's medical record lists the client's religion as Jehovah's Witness. What action should the nurse take? A) Prepare to infuse multiple units of fresh frozen plasma B) Inform the client of the critical need for a blood transfusion C) Clarify the clients wishes about receiving blood products D) Obtain consent from the family to infuse packed red blood cells

C) Clarify the clients wishes about receiving blood products

A new mother, who is a lacto-ovo vegetarian, plans to breastfeed her infant. What information should the nurse provide prior to discharge. A) Avoid using lanolin-based nipple cream or ointment B) Offer iron-fortified supplemental formula daily. C) Continue prenatal vitamins with B12 while breast feeding D) Weight the baby weekly to evaluate the newborn's growth

C) Continue prenatal vitamins with B12 while breast feeding

The nurse is performing a newborn assessment. Which symptoms if present in newborn, would indicate respiratory distress? A) Abdominal breathing with synchronous chest movement B) Shallow and irregular respirations C) Flaring of the nares D) Respiratory rate of 50 breaths per minute

C) Flaring of the nares

The nurse is caring for a postpartum client who is complaining of severe pain and a feeling of pressure in her perineum. Her fundus is firm, and she has a moderate lochia flow. On inspection the nurse finds that a perineal hematoma is beginning to form. Which assessment findings show the nurse obtain first? A) Abdominal contour and bowel sounds B) Hemoglobin and hematocrit C) Heart rate and blood pressure D) Urinary output and IV fluid intake

C) Heart rate and blood pressure

At 0600 while admitting a woman for a scheduled repeat Caesarean section (C-section), the client tells the nurse that she drank a cup of coffee at 0400 because she wanted to avoid getting a headache. Which action should the nurse take first? A) Contact the client's obstetrician B) Ensure preoperative lab results are available C) Inform the anesthesia care provider D) Start prescribed IV with Lactated Ringer's

C) Inform the anesthesia care provider

A woman in her third trimester of pregnancy has been in active labor for the past 8 hours and cervix dialed 3cm. The nurse's assessment findings and electronic fetal monitoring (EFM) are consistent with hypotonic dystocia, and the healthcare provider prescribes an oxytocin drip. Which data is most important for the nurse to monitor? A) Clients hourly blood pressure B) Preparation for emergency cesarean birth C) Intensity, interval, and length of contractions D) Checking the perineum for bulging

C) Intensity, interval, and length of contractions

A diabetic client delivers a full-term large for gestation- age (LGA) infant who is jittery action should the nurse take first? A) Administer oxygen B) Feed the infant glucose water (10%) C) Obtain a blood glucose level D) Decrees environment stimuli

C) Obtain a blood glucose level

One hour after delivery the nurse is unable to palpate the uterine fundus of a client who had an epidural and notes a large amount of lochia on the perineal pad. The nurse massages at the umbilicus and obtains current vital signs. Which intervention should the nurse implement next. A) Document number of pad changes in the last hour B) Provide bedpans to void if unable to ambulate C) Palpate the supra pubic area for bladder distention D) Increases the rate of the oxytocin infusion

C) Palpate the supra pubic area for bladder distention

A client at 20 weeks gestation comes to antepartum clinic complaining of vaginal warts (human papilloma virus HPV). What information should the nurse provide this client? A) Termination of the pregnancy should be considered B) Pregnancy complication are not linked to HPV C) This client should be treated with acyclovir (Zovirax) D) The client should be treated with penicillin G.

C) This client should be treated with acyclovir (Zovirax)

A client who delivered a healthy newborn an hour ago asks the nurse when she can go home. Which information is most important for the nurse to provide the client? A) When ambulating to void does not cause dizziness B) After the vitamin K injection is given to the baby C) When there is no significant vaginal bleeding. D) After the baby no longer demonstrates acrocyanosis

C) When there is no significant vaginal bleeding

The nurse is caring for a newborn infant who was recently diagnosed with congenital heart defect. Which assessment finding warrants immediate intervention by the nurse? A. Sweating during feedings B. Weak peripheral pulse C. Bluish tinge to the tongue D. Increased respiratory rate

C. Bluish tinge to the tongue

The father of a 3- day- old infant who is breast feeding calls the postpartum help line to report that his wife is acting strangely. She is irritable, cannot cope with the baby, and frequently cried for no apparent reason. What information is most important for the nurse to provide to this father? A. A fluctuation in hormones in the early postpartum period can cause mood changes B. Recommend giving supplemental bottle feedings to the baby between breast feeding C. Contact the clinic if the behaviors continue for more than two weeks or become worse D. Tell the father to count the newborns number of soiled diapers over the next few days

C. Contact the clinic if the behaviors continue for more than two weeks or become worse

The nurse is scheduling a client with gestational diabetes for an amniocentesis because the fetus has an estimated weight of 8 pounds at 36- weeks gestation. This amniocentesis is being performed to obtain which information? A. Presence of a neural tube defect B. Gender of the fetus C. Fetal lung maturity D. Chromosomal abnormalities

C. Fetal lung maturity

One week after missing her menstrual period, a woman performs an OTC pregnancy test and it is positive. Which hormone is responsible for producing the positive result? A. Human placental lactogen B. Gonadotrophin-releasing hormone C. Human chorionic gonadotrophin D. Prostaglandin E2 Alpha

C. Human chorionic gonadotrophin

One hour after delivery, the nurse is unable to palpate the uterine fundus of a client who had an epidural and notes a large amount of lochia on the perineal pad. The nurse massages at the umbilicus and obtains current vital signs. Which intervention should the nurse implement next? A. Document number of pad changes in the last hour B. Increase the rate of the oxytocin infusion C. Palpate the suprapubic area for bladder distention D. Provide bedpan to void if unable to ambulate

C. Palpate the suprapubic area for bladder distention

The nurse is assessing a postpartum client who delivered a 10-pound infant vaginally two hours ago. The clients fundus is 2 fingerbreadths above the umbilicus, deviated to the right side, and boggy. After the client voids 250 ml of urine using a bedpan, what action should the nurse implement? A. Re-evaluate the client in 15 minutes B. Assist the client to the bathroom to void C. Palpate the suprapubic region for distention D. Encourage the client to breastfeed

C. Palpate the suprapubic region for distention

The health care provider hands a newborn to the nurse after a vaginal delivery. What action is most important for the nurse to implement? A. Allow the mother to touch the infant B. Complete a physical assessment C. Place the infant under a warming unit D. Determine the APGAR score

C. Place the infant under a warming unit

A primigravida at 36-weeks gestation, who is Rh negative, experienced abdominal trauma in a motor vehicle collision. Which assessment finding is most important for the nurse to report to the health care provider? A. Fetal heart rate of 162 beats/minute B. Trace of protein in the urine C. Positive fetal hemoglobin test D. Mild contractions every 10 minutes

C. Positive fetal hemoglobin test

A multiparous woman at 38-weeks gestation with a history of rapid progression of labor is admitted for induction due to signs and symptoms of preeclampsia. One hour after the Pitocin infusion is initiated, she complains of a headache. Her contractions are occurring every 1 to 2 minutes, lasting 60 to 75 seconds, and a vaginal exam indicates that her cervix is 90% effaced and dilated to 6 cm. What intervention is most important for the nurse to implement? A. Turn the client on her left side B. Discontinue the Pitocin infusion C. Prepare for immediate delivery D. Measure deep tendon reflexes

C. Prepare for immediate delivery

A client who is receiving oxytocin (Pitocin) to augment early labor begins to experience hyper systolic or tetanic contractions with variable fetal heart decelerations. Which action should the nurse implement? A. Reposition the fetal monitor transducers B. Alert the charge nurse to the patient's condition C. Turn off the Pitocin infusion D. Decrease the rate of the Pitocin infusion

C. Turn off the Pitocin infusion

Vaginal prostaglandin gel is used to induce labor for a woman who is at 42 weeks gestation. Thirty minutes after insertion of the gel, the client complains of vaginal warmth, and is experiencing 90 second contractions with fetal heart rate decelerations. What action should the nurse implement first? A. Notify the hcp B. Assess the maternal vital signs C. Turn to a side-lying position D. Increase the IV infusion rate

C. Turn to a side-lying position

A client at 30-weeks' gestation, complaining of pressure over the pubic area, is admitted for observation. She is contracting irregularly and demonstrates underlying uterine irritability. Vaginal examination reveals that her cervix is closed, thick, and high. Based on these data, which intervention should the nurse implement first?

C.Obtain a specimen for urine analysis. Obtaining a urine analysis should be done first because preterm clients with uterine irritability and contractions are often suffering from a urinary tract infection, and this should be ruled out first.

A client with NO prenatal care arrives at the labor unit screaming, "The baby is coming!" The nurse performs a vaginal examination that reveals the cervix is 3 centimeters dilated and 75% effaced. What additional information is most important for the nurse to obtain?

Date of last normal menstrual period. Evaluating the gestation of the pregnancy takes priority. If the fetus is preterm and the fetal heart pattern is reassuring, the healthcare provider may attempt to prolong the pregnancy and administer corticosteroids to mature the lungs of the fetus.

What is Epogen for?

Changes an apical heart rate from the 180s to the 140s. Epogen, given to prevent or treat anemia, stimulates erythropoietin production, resulting in an increase in RBCs. Since the body has not had to compensate for anemia with an increased heart rate, changes in heart rate from high to normal is one indicator that Epogen is effective

A full-term infant is admitted to the newborn nursery and, after careful assessment, the nurse suspects that the infant may have an esophageal atresia. Which symptoms is this newborn likely to have exhibited?

Choking, coughing, and cyanosis. the "3 Cs" of esophageal atresia caused by the overflow of secretions into the trachea.

A client at 28-weeks gestation calls the antepartum clinic and states that she is experiencing a small amount of vaginal bleeding which she describes as bright red. She further states that she is not experiencing any uterine contractions or abdominal pain. What instruction should the nurse provide?

Come to the clinic today for an ultrasound Third trimester painless bleeding is characteristic of a placenta previa. Bright red bleeding may be intermittent, occur in gushes, or be continuous. Rarely is the first incidence life-threatening, nor cause for hypovolemic shock. Diagnosis is confirmed by transabdominal ultrasound.

A pregnant client with mitral stenosis Class III is prescribed complete bedrest. The client asks the nurse, "Why must I stay in bed all the time?" Which response is best for the nurse to provide this client?

Complete bedrest decreases oxygen needs and demands on the heart muscle tissue. To help preserve cardiac reserves, the woman may need to restrict her activities and complete bedrest is often prescribed.

One hour after giving birth to an 8-pound infant, a client's lochia rubra has increased from small to large and her fundus is boggy despite massage. The client's pulse is 84 beats/minute and blood pressure is 156/96. The healthcare provider prescribes Methergine 0.2 mg IM × 1. What action should the nurse take immediately?

Contact the healthcare provider and question the prescription Methergine is contraindicated for clients with elevated blood pressure, so the nurse should contact the healthcare provider and question the prescription

When assessing a pregnant woman AT 39-weeks gestation who is admitted to labor and delivery which finding is most important to report to the health care provider? A) + proteinuria B) 130/70 blood pressure C) + pedal edema D) 101.2 oral temperature

D) 101.2 oral temperature

The nurse is caring for a laboring client who is GBS+ (Group B streptococcus). Which immediate treatment is indicated for this client? A) Administration of Pitocin B) Artificial rupture of the membrane C) Amnioinfusion for the baby D) Administration of antibodies

D) Administration of antibodies

The nurse examines a client who is admitted in active labor and determines the cervix is 3cm dilated 50% effaced, and the presenting part is at 0 stations. An hour later, she tells the nurse that she wants to go to the bathroom. Which action should the nurse implement first. A) Check the pH of the vaginal fluid B) Review the fetal heart rate pattern C) Palpate the client's bladder D) Determine cervical dilation

D) Determine cervical dilation

At 6 weeks gestation the rubella titer of a client medication indicates she is non- immune. When is the best time to administer a rubella vaccine to this client? A) After the client stops breastfeeding B) Immediately, at 6-weeks gestation to protect fetus C) After the client reaches 20-weeks gestations D) Early postpartum within 72 hours of delivery

D) Early postpartum within 72 hours of delivery

pregnant, homeless woman who has received no prenatal care presents to the clinic in her third trimester because she is having vaginal bleeding but reports that she is not in pain. Ultrasound reveals a placenta previa. Which actions should the nurse implement? A) Schedule weekly prenatal appointments B) Contact social services for a temporary shelter C) Obtain a hemoglobin and hematocrit level D) Have the client transported to the hospital

D) Have the client transported to the hospital

When teaching a gravid client how to perform kick (fetal movement) counts which instruction should the nurse includes. A) Exercise for 15 before starting the counting to help increase fetal movement B) Count the movements once daily for one hour, before breakfast C) Avoid caffeinated drinks for 24 hours before conducting the kick test. D) If 10 kicks are not felt within 1hr, drink orange juice and count for another hour.

D) If 10 kicks are not felt within 1hr, drink orange juice and count for another hour.

A 6 weeks gestation, the rubella titer of a client indicates she is non-immune. When is the best time to administer a rubella vaccine to this client? a. immediately, at six weeks gestation to protect this fetus b. early postpartum within 72 hours of delivery c. after the client stops breastfeeding d. after the client reaches 20-weeks' gestation

b. early postpartum within 72 hours of delivery

A term multigravida, who is receiving oxytocin for labor augmentation is requesting pain medication. Review of the clients record indication that she was medicated 30minutes ago with butorphanol (Stadol) 2mg and promethazine (Phenergan) 25mg IV push. Vaginal examination reveals that the client cervical dilation is 3cm, 70% effaced, and at a 0 station. What action should the nurse implement? A) Discontinue the Pitocin infusion B) Medicate the client with an additional 1mg of Stadol IV push C) Notify the healthcare provider D) Instruct the client to use deep breathing during contraction

D) Instruct the client to use deep breathing during contraction

Following the vaginal delivery of a 10-pound infant, the nurse assesses a new mothers vaginal bleeding and finds that she has saturated two pads in 30minutes and has a boggy uterus. What action should the nurse implement first? A) Have the client empty her bladder B) Inspect the perineum for lacerations C) Increase oxytocin IV infusion D) Perform fundal massage until firm

D) Perform fundal massage until firm

An obviously pregnant woman walks into the hospital's emergency department entrance shouting. "Help me! Help me! My baby is coming! I'm so afraid!" The nurse determines if delivery is indeed imminent, what action is most important for the nurse to take? A) Determines the gestational age of fetus B) Assess the amount and color of the amniotic fluid C) Obtain peripheral IV access and begin administration of IV fluids D) Provide clear concise instructions in a calm, deliberate manner

D) Provide clear concise instructions in a calm, deliberate manner

After delivery of a normal infant, the mother tells the nurses that she would like to use oral contraceptive. Which finding in the client's health history is a contraindication of the use of contraceptives? A) Previously used intrauterine device (IUD) B) Reported history of stroke within family C) Diagnosed with diabetes mellitus 2 years ago D) Smoked cigarettes prior to becoming pregnant

D) Smoked cigarettes prior to becoming pregnant

The nurse is discussing involution with a post-partum client. Which statement best indicates that the client understands the effect of breastfeeding on the resumption of menstrual cycle? A. "My period will most likely return in 6 to 8 months" B. "I should expect my period to return in 6 to 8 weeks" C. "My period started as soon as the baby was born" D. "While I am breastfeeding, my period may be delayed"

D. "While I am breastfeeding, my period may be delayed"

While caring for a laboring client on continuous fetal monitoring, the nurse notes a fetal heartrate pattern that falls and rises abruptly with a "V" shaped appearance. What action should the nurse take first? A. Prepare for a potential cesarean B. Allow the client to begin pushing C. Administer oxygen at 10/L by mask D. Change the maternal position

D. Change the maternal position

The nurse is counseling a client who is at 6 weeks gestation and is experiencing morning sickness but does not want to take any drugs for this discomfort. Which herbal supplement is likely to help this client with the nausea she is experiencing? A. Gingko B. Chamomile C. Peppermint D. Ginger

D. Ginger

During the admission of a newborn, the nurse identifies a localized swelling that does not cross the suture line on the posterior area of the parietal bone. What action should the nurse implement? A. Assess neurological vital signs every 4 hours B. Apply direct pressure to the caput succedaneum (THIS ONE CROSSES THE SUTURE LINES) C. Submit a request for a stat CT scan of the head D. Notify the pediatrician of the cephalohematoma (THIS ONE DOES NOT CROSS THE SL & IS MORE CRITICAL)

D. Notify the pediatrician of the cephalohematoma (THIS ONE DOES NOT CROSS THE SL & IS MORE CRITICAL)

The nurse is assessing a newborn who was precipitously delivered at 38 weeks gestation. The newborn is tremulous, tachycardic, and hypertensive. Which assessment action is most important for the nurse to implement? A. Determine reactivity of neonatal reflexes B. Perform gestational age assessment C. Weight and measure the newborn D. Obtain a drug screen for cocaine

D. Obtain a drug screen for cocaine

A newborn infant is receiving immunization prior to discharge. Which action should the nurse implement? A. Give the first dose of the vaccine for Rotavirus if any siblings have diarrhea now B. Ask the mother if she wants the infant immunized for Haemophilus influenza C. Prepare the first dose for Diphtheria, tetanus toxoid and acellular pertussis (DTap) D. Obtain signed consent from the mother for administration of hepatitis B vaccine

D. Obtain signed consent from the mother for administration of hepatitis B vaccine

The nurse is assessing a 35-week primigravida with a breech presentation who is experiencing moderate uterine contraction every 3-5 minutes. During the examination the client tells the nurse, "I think my water just broke". Inspection of the perineal area reveals the umbilical cord protruding from the vagina. After activating the call bell system for assistance, what intervention should the nurse implement? A. Administer oxygen at 10 liters via face mask B. Don gloves and push the cord back into the vagina C. Wrap the umbilical cord with sterile gauze D. Position the client into a knee-chest position

D. Position the client into a knee-chest position

A multiparous client at 38- weeks gestation is admitted to labor and delivery with a compliant of contractions 5 minutes apart. While the client is in the bathroom changing into a hospital gown, the nurse hears a baby crying. What action should the nurse take first? A. Inspect the client's perineum B. Turn on the infant warmer C. Notify a healthcare provider D. Push the call light for help

D. Push the call light for help

A client who is in active labor is receiving magnesium sulfate and begin to experience slurred speech and decreased reflexes. Which action should the nurse implement first? A. Obtain a serum magnesium level B. Measure the clients hourly urinary output C. Provide an emesis basin for vomiting D. Turn off the magnesium sulfate infusion

D. Turn off the magnesium sulfate infusion

At 34- weeks gestation, a primigravida is assessed at her bimonthly clinic visits, which assessment finding is important for the nurse to report to the hcp? A. Increased appetite B. Fetal heart rate of 110 beats/minute C. Fundus below the xiphoid D. Weight gain of 7 pounds

D. Weight gain of 7 pounds

A client who delivered a healthy newborn an hour ago asks the nurse when she can go home. Which information is most important for the nurse to provide the client? A. When ambulating to void does not cause dizziness B. After the vitamin K injection is given to the baby C. After the baby no longer demonstrates acrocyanosis D. When there is no significant vaginal bleeding

D. When there is no significant vaginal bleeding

A woman with Type 2 diabetes mellitus becomes pregnant, and her oral hypoglycemic agents are discontinued. Which intervention is most important for the nurse to implement?

Describe diet changes that can improve the management of her diabetes Diet modifications are effective in managing Type 2 diabetes during pregnancy and describing the necessary diet changes is the most important intervention for the nurse to implement with this client.

A client at 32-weeks' gestation comes to the prenatal clinic with complaints of pedal edema, dyspnea, fatigue, and a moist cough. Which question is most important for the nurse to ask this client?

Do you have a history of rheumatic fever? Clients with a history of rheumatic fever may develop mitral valve prolapse, which increases the risk for cardiac decompensation due to the increased blood volume that occurs during pregnancy, so obtaining information about this client's health history is a priority.

The nurse is caring for a woman with a previously diagnosed heart disease who is in the second stage of labor. Which assessment findings are of greatest concern?

Edema, basilar rales, and an irregular pulse This indicates cardiac decompensation and requires immediate intervention

A 24-hour-old newborn has a pink papular rash with vesicles superimposed on the thorax, back, and abdomen. What action should the nurse implement? documented the findings in the infant's record

Erythema toxicum (or erythema neonatorum) is a newborn rash that is commonly referred to as "flea bites," but is a normal finding that is documented in the infant's record.

A 28-year-old client in active labor complains of cramps in her leg. What intervention should the nurse implement?

Extend the leg and dorsiflex the foot Dorsiflexing the foot by pushing the sole of the foot forward or by standing (if the client is capable) and putting the heel of the foot on the floor is the best means of relieving leg cramps

A client at 38- weeks' gestation complaints of severe abdominal pain. Upon the nurse notes that the abdomen is rigid. a. Placenta previa b. Oligoamnios c. Abruptio placenta d. Chorioamnionitis

c. Abruptio placenta

A primigravida at 40-weeks gestation is receiving oxytocin (Pitocin) to augment labor. Which adverse effect should the nurse monitor for during the infusion of Pitocin?

Fetal tachycardia Pitocin causes the uterine myofibril to contract, so unless the infusion is closely monitored, the client is at risk for hyperstimulation which can lead to tetanic contractions, uterine rupture, and fetal distress or demise.

The healthcare provider prescribes terbutaline (Brethine) for a client in preterm labor. Before initiating this prescription, it is most important for the nurse to assess the client for which condition?

Gestational diabetes. The nurse should evaluate the client for gestational diabetes because terbutaline (Brethine) increases blood glucose levels.

The nurse is teaching care of the newborn to a group of prospective parents and describes the need for administering antibiotic ointment into the eyes of the newborn. Which infectious organism will this treatment prevent from harming the infant?

Gonorrhea Erythromycin ointment is instilled into the lower conjunctiva of each eye within 2 hours after birth to prevent ophthalmic neonatorum, an infection caused by gonorrhea, and inclusion conjunctivitis, an infection caused by chlamydia. The infant may be exposed to these bacteria when passing through the birth canal.

The nurse instructs a laboring client to use accelerated-blow breathing. The client begins to complain of tingling fingers and dizziness. What action should the nurse take?

Have the client breathe into her cupped hands Tingling fingers and dizziness are signs of hyperventilation (blowing off too much carbon dioxide). Hyperventilation is treated by retaining carbon dioxide. This can be facilitated by breathing into a paper bag or cupped hands.

A 38-week primigravida who works as a secretary and sits at a computer for 8 hours each day tells the nurse that her feet have begun to swell. Which instruction would be most effective in preventing pooling of blood in the lower extremities?

Move about every hour Pooling of blood in the lower extremities results from the enlarged uterus exerting pressure on the pelvic veins. Moving about every hour will straighten out the pelvic veins and increase venous return.

Which maternal behavior is the nurse most likely to see when a new mother receives her infant for the first time?

Her arms and hands receive the infant and she then traces the infant's profile with her fingertips. Attachment/bonding theory indicates that most mothers will demonstrate behaviors described in during the first visit with the newborn, which may be at delivery or later.

What is Alpha-fetoprotein (AFP)?

High levels of this in the amniotic fluid help confirm the diagnosis of NTD such as spina bifida, anencephaly, or an abdominal wall defect

The parents of a male newborn have signed an informed consent for circumcision. What priority intervention should the nurse implement upon completion of the circumcision? a. give a PRN dose of liquid acetaminophen b. wrap the infant in warm receiving blankets c. place petrolatum gauze dressings on the site * d. offer a pacifier dipped in glucose water

c. place petrolatum gauze dressings on the site *

Total placental previa

If you have placenta previa, it means that your placenta is lying unusually low in your uterus, next to or covering your cervix... .. (You may also hear the term "partial previa," which refers to a placenta that covers part of the cervical opening once the cervix starts to dilate.) The egg implants near the bottom of the cervix, the placenta forms around it and the placenta blocks the opening so the baby cannot come out without a C-section unless the placenta moves on its own.

Define Pseudo strabismus

The false appearance of crossed eyes. When the eyes are actually crossed or not completely aligned with one another it is called strabismus. Pseudo strabismus generally occurs in infants and toddlers whose facial features are not fully developed.

Immediately after birth a newborn infant is suctioned, dried, and placed under a radiant warmer. The infant has spontaneous respirations and the nurse assesses an apical heart rate of 80 beats/minute and respirations of 20 breaths/minute. What action should the nurse perform next?

Initiate positive pressure ventilation The nurse should immediately begin positive pressure ventilation because this infant's vital signs are not within the normal range, and oxygen deprivation leads to cardiac depression in infants. (The normal newborn pulse is 100 to 160 beats/minute and respirations are 40 to 60 breaths/minute.)

A client who is attending antepartum classes asks the nurse why her healthcare provider has prescribed iron tablets. The nurse's response is based on what knowledge?

It is difficult to consume 18 mg of additional iron by diet alone. Consuming enough iron-containing foods to facilitate adequate fetal storage of iron and to meet the demands of pregnancy is difficult so iron supplements are often recommended.

A client who is in the second trimester of pregnancy tells the nurse that she wants to use herbal therapy. Which response is best for the nurse to provide?

It is important that you want to take part in your care. The emphasis of alternative and complementary therapies, such as herbal therapy, is that the client is viewed as a whole being, capable of decision-making and an integral part of the health care team, so recognizes the client's request.

Which nursing intervention is most helpful in relieving postpartum uterine contractions or "afterpains?"

Lying prone with a pillow on the abdomen Lying prone keeps the fundus contracted and is especially useful with multiparas, who commonly experience afterpains due to lack of uterine tone.

Define Milia

Milia are tiny white bumps that appear across a baby's nose, chin or cheeks. Milia are common in newborns but can occur at any age.

The nurse assesses a client admitted to the labor and delivery unit and obtains the following data: dark red vaginal bleeding, uterus slightly tense between contractions, BP 110/68, FHR 110 beats/minute, cervix 1 cm dilated and uneffaced. Based on these assessment findings, what intervention should the nurse implement?

Monitoring bleeding from IV sites Monitoring bleeding from peripheral sites is the priority intervention. This client is presenting with signs of placental abruption. Disseminated intravascular coagulation (DIC) is a complication of placental abruptio, characterized by abnormal bleeding.

A 30-year-old gravida 2, para 1 client is admitted to the hospital at 26-weeks' gestation in preterm labor. She is given a dose of terbutaline sulfate (Brethine) 0.25 mg subcutaneous. Which assessment is the highest priority for the nurse to monitor during the administration of this drug?

Monitoring maternal and fetal heart rates is most important when terbutaline is being administered. Terbutaline acts as a sympathomimetic agent that stimulates both beta 1 receptors (causing tachycardia, a side effect of the drug) and stimulation of beta 2 receptors (causing uterine relaxation, a desired effect of the drug).

non-stress test (NST) - how to pass?

Must have fetal heart rate of 15 beats above normal heart rate for 15 seconds two times or more over 20 minutes. If the baby is asleep, wake it up. A Fetal acoustic stimulator works best. (FAST)

The nurse identifies crepitus when examining the chest of a newborn who was delivered vaginally. Which further assessment should the nurse perform?

Observe for an asymmetrical Moro (startle) reflex The most common neonatal birth trauma due to a vaginal delivery is fracture of the clavicle. Although an infant may be asymptomatic, a fractured clavicle should be suspected if an infant has limited use of the affected arm, malposition of the arm, an asymmetric Moro reflex, crepitus over the clavicle, focal swelling or tenderness, or cries when the arm is moved.

Twenty minutes after a continuous epidural anesthetic is administered, a laboring client's blood pressure drops from 120/80 to 90/60. What action should the nurse take?

Place woman in a lateral position The nurse should immediately turn the woman to a lateral position, place a pillow or wedge under the right hip to deflect the uterus, increase the rate of the main line IV infusion, and administer oxygen by face mask at 10-12 L/min. If the blood pressure remains low, especially if it further decreases, the anesthesiologist/healthcare provider should be notified immediately.

An off-duty nurse finds a woman in a supermarket parking lot delivering an infant while her husband is screaming for someone to help his wife. Which intervention has the highest priority?

Put the newborn to breast Putting the newborn to breast will help contract the uterus and prevent a postpartum hemorrhage--this intervention has the highest priority.

A client receiving epidural anesthesia begins to experience nausea and becomes pale and clammy. What intervention should the nurse implement first?

Raise the foot of the bed These symptoms are suggestive of hypotension which is a side effect of epidural anesthesia. Raising the foot of the bed will increase venous return and provide blood to the vital areas. Increasing the IV fluid rate using a balanced non-dextrose solution and ensuring that the client is in a lateral position are also appropriate interventions, and then checking the patient's blood pressure.

The nurse is providing discharge teaching for a client who is 24 hours postpartum. The nurse explains to the client that her vaginal discharge will change from red to pink and then to white. The client asks, "What if I start having red bleeding AFTER it changes?" What should the nurse instruct the client to do?

Reduce activity level and notify the healthcare provider. Lochia should progress in stages from rubra (red) to serosa (pinkish) to alba (whitish), and not return to red. The return to rubra usually indicates subinvolution or infection. If such a sign occurs, the mother should notify the clinic/healthcare provider and reduce her activity to conserve energy

What does a diaphragmatic hernia look like?

Scaphoid abdomen and anorexia

A couple, concerned because the woman has not been able to conceive, is referred to a healthcare provider for a fertility workup and a hysterosalpingography is scheduled. Which post procedure complaint indicates that the fallopian tubes are patent?

Shoulder pain If the tubes are patent (open), pain is referred to the shoulder from a sub diaphragmatic collection of peritoneal dye/gas.

After each feeding, a 3-day-old newborn is spitting up large amounts of Enfamil® Newborn Formula, a nonfat cow's milk formula. The pediatric healthcare provider changes the neonate's formula to Similac® Soy Isomil® Formula, a soy protein isolate based infant formula. What information should the nurse provide to the mother about the newly prescribed formula?

Similac® Soy Isomil® Formula is a soy protein-based formula that contains sucrose The nurse should explain that the newborn's feeding intolerance may be related to the lactose found in cow's milk formula and is being replaced with the soy-based formula that contains sucrose, which is well-tolerated in infants with milk allergies and lactose intolerance.

A 26-year-old, gravida 2, para 1 client is admitted to the hospital at 28-weeks gestation in preterm labor. She is given 3 doses of terbutaline sulfate (Brethine) 0.25 mg subcutaneously to stop her labor contractions. The nurse plans to monitor for which primary side effect of terbutaline sulfate?

Tachycardia and a feeling of nervousness Terbutaline sulfate (Brethine), a beta-sympathomimetic drug, stimulates beta-adrenergic receptors in the uterine muscle to stop contractions. The beta-adrenergic agonist properties of the drug may cause tachycardia, increased cardiac output, restlessness, headache, and a feeling of "nervousness".

A client with gestational hypertension is in active labor and receiving an infusion of magnesium sulfate. Which drug should the nurse have available for signs of potential toxicity?

The antidote for magnesium sulfate is calcium gluconate, which should be readily available if the client manifest signs of toxicity.

The nurse is assessing a client who is having a non-stress test (NST) at 41-weeks gestation. The nurse determines that the client is not having contractions, the fetal heart rate (FHR) baseline is 144 bpm, and no FHR accelerations are occurring. What action should the nurse take?

The client should be asked if she has felt the fetus move. An NST is used to determine fetal well-being and is often implemented when post maturity is suspected. A "reactive" NST occurs if the FHR accelerates 15 bpm for 15 seconds in response to the fetus' own movement and is "nonreactive" if no FHR acceleration occurs in response to fetal movement.

What did Nurse theorist Reva Rubin describe?

The initial postpartum period as the "taking-in phase," which is characterized by maternal reliance on others to satisfy the needs for comfort, rest, nourishment, and closeness to families and the newborn.

While breastfeeding, a new mother strokes the top of her baby's head and asks the nurse about the baby's swollen scalp. The nurse responds that the swelling is caput succedaneum. Which additional information should the nurse provide this new mother?

The scalp edema will subside in a few days after birth Caput succedaneum is edema of the fetal scalp that crosses over the suture lines and is caused by pressure on the fetal head against the cervix during labor; it subsides in a few days after birth without treatment

Urinary output must be monitored when administering magnesium sulfate and should be at least 30 ml per hour. (The therapeutic level of magnesium sulfate for a PIH client is 4.8 to 9.6 mg/dl.) What is the therapeutic level of magnesium sulfate?

The therapeutic level of magnesium sulfate for a PIH client is 4.8 to 9.6 mg/dl. What does it help prevent? helps prevent seizures What indicates toxic levels? 3 Respiratory rate below 12 indicates toxic effects. Urine output of less than 100 ml/4 hours Absent DTRs

Define Subarachnoid hematoma

When blood leaks into the space between two of the membranes that surround the brain. It is usually caused by a ruptured aneurysm ............. The bleeding occurs in the arteries just below the arachnoid membrane and above the pia mater, just below the surface of the skull. Do nothing, it will most likely re-absorb on its own over time or they become brain damaged.

Client teaching is an important part of the maternity nurse's role. Which factor has the greatest influence on successful teaching on the gravid client? a. The client's readiness to learn b. The client's educational background c. The order in which the information is presented d. The extent to which the pregnancy was planned

When teaching any client, readiness to learn (A) is the most important criterion. For example, the client with severe morning sickness in the first trimester may not be "ready to learn" about ways to relieve morning sickness.

Twenty-four hours after admission to the newborn nursery, a full-term male infant develops localized edema on the right side of his head. The nurse knows that, in the newborn, an accumulation of blood between the periosteum and skull which does not cross the suture line is a newborn variation known as...

a cephalohematoma, caused by forceps trauma and may last up to 8 weeks. Cephalohematoma, a slight abnormal variation of the newborn, usually arises within the first 24 hours after delivery. Trauma from delivery causes capillary bleeding between the periosteum and the skull.

Define Biophysical profile (BPP)

a prenatal ultrasound evaluation of fetal well-being involving a scoring system, with the score being termed Manning's score. It scores the fetuses responses to stimuli, is non-invasive, and based on acute and chronic markers of fetal disease. Measures: • Fetal breathing movements • Fetal movements • Fetal tone • Amniotic fluid index (AFI)

Hysterosalpingography (HSG) AKA uterosalpingography

a radiologic procedure to investigate the patency of the fallopian tubes. It entails the injection of a radio-opaque material into the cervical canal and usually fluoroscopy with image intensification.

A client who suspects she is pregnant tells the nurse she has a peptic ulcer that is being treated with misoprostol (Cytotec), a synthetic prostaglandin C drug, how should the nurse respond? a. "You may be at risk for having a spontaneous miscarriage" b. "You may have an increased chance of having preeclampsia" c. "This medication will have no effect on your unborn child" d. "You may experience postpartum hemorrhaging after delivery"

a. "You may be at risk for having a spontaneous miscarriage"

The father of a 3-day old infant who is breast feeding calls the postpartum help line to report that his wife is acting strangely. She is irritable, cannot cope with the baby, and frequently cries for no appeared reason. What information is most important for the nurse to provide the father? a. Contact the clinic if the behaviors continue for more than two weeks or becomes worse b. Tell the father count the newborns number of soiled diapers over the next few days. c. A fluctuation in hormones in the early postpartum period can cause mood changes. d. Recommend giving supplemental bottle feedings to the baby between breast feeding.

a. Contact the clinic if the behaviors continue for more than two weeks or becomes worse

When planning care for a laboring client, the nurse identifies the need to withhold solids food while the client is in labored. What is the most important reasons for this nursing intervention?

a. An increased risk of aspiration can occur if general anesthesia is needed

The nurse is teaching a woman how to use her basal body temperature (BBT) pattern as a tool to assist her in conceiving a child. Which temperature pattern indicates the occurrence of ovulation, and there for, the best time for intercourse to ensure conception? a. Between the time the temperature falls and rises b. Between 36 and 48 hours after the temperature rises c. When the temperature falls and remains low for 36 hours d. Within 72 hours before the temperature falls

a. Between the time the temperature falls and rises In most women, the BBT drops slightly 24 to 36 hours before ovulation and rises 24 to 72 hours after ovulation, when the corpus luteum of the ruptured ovary produces progesterone. Therefore, intercourse between the time of the temperature fall and rise (A) is the best time for conception.

While caring for a laboring client on continuous fetal monitoring. The nurse notes an FHR pattern that falls and rises abruptly with a "V" shaped appearance. What action should the nurse take first?

a. Change the maternal position

A multigravida client arrives at the labor and delivery unit and tells the nurse that her bag of water has broken. The nurse identifies the presence of meconium fluid on the perineum and determines the fetal heart rate is between 140 to 150 beats/minute. What action should the nurse implement next? a. Complete a sterile vaginal exam b. Take maternal temperature every 2 hours c. Prepare for an immediate cesarean birth d. Obtain sterile suction equipment

a. Complete a sterile vaginal exam A vaginal exam (A) should be performed after the rupture of membranes to determine the presence of a prolapsed cord.

A pregnant client with mitral stenosis Class III is prescribed complete bedrest. The client asks the nurse, "Why must I stay in bed all the time?" Which response is best for the nurse to provide this client? a. Complete bedrest decreases oxygen needs and demands on the heart muscle b. We want your baby to be healthy, and this is the only way we can make sure that will happen again c. I know you're upset. Would you like to talk about somethings you could so while in bed? d. Labor is difficult, and you need to use this time to rest before you have to assume all child-caring duties

a. Complete bedrest decreases oxygen needs and demands on the heart muscle To help preserve cardiac reserves, the woman may need to restrict her activities and complete bedrest is often prescribes (A).

The father of a 3-day old infant who is breast feeding calls the postpartum help line to report that his wife is acting strangely. She is irritable, cannot cope with the baby, and frequently cries for no apparent reason. What information is most important for the nurse to provide this father?

a. Contact the clinic if the behaviors continue for more than two weeks or become worse.

A healthcare provider informs the charge nurse of a labor and delivery unit that a client is coming to the unit with suspected abruptio placentae. What findings should the charge nurse expect the client to demonstrate? (Select all that apply) a. Dark, red vaginal bleeding b. Lower back pain c. Premature rupture of membranes d. Increased uterine irritability e. Bilateral pitting edema f. A rigid abdomen

a. Dark, red vaginal bleeding d. Increased uterine irritability f. A rigid abdomen The symptoms of abruptio placentae include dark red vaginal bleeding (A), increased uterine irritability (D), and a rigid abdomen (F).

The home health nurse visits a client who delivered a full-term baby three days ago. The mother reports that the infant is waking up every 2 hours to bottle feed. The nurse notes white, curl-like patches on the newborns oral mucous membranes. What action should the nurse implement? a. Discuss the need for medication to treat curl-like oral patches b. Suggest switching the infant's formula c. Assess the baby's blood glucose level d. Remind mother not put the baby to bed with a propped bottle

a. Discuss the need for medication to treat curl-like oral patches

A client who is anovulatory and has hyperprolactinemia is being treated for infertility with metformin, menotropins (Repronex, menopur), and human chorionic gonadotropin(hCG). Which side effects should the nurse tell the client to report immediately? a. Episodes of headache and irritability b. Nausea and vomiting c. Rapid increase in abdominal girth d. Persistent daytime fatigue

a. Episodes of headache and irritability

The nurse is performing a newborn assessment. Which symptoms if present in newborn, would indicate respiratory distress?

a. Flaring of the nares

Calculated by Naegele's rule, a primigravida client is at 28 weeks gestation. She is moderately obese and carrying twins and the nurse measures her fundal height at 27 cm. During the previous visit 3 weeks ago, the fundal height measured at 28 cm. Based on these findings, what should the nurse conclude?

a. Fundal height measurement may indicate intrauterine growth retardation

One week after missing her menstrual period, a woman performs an OTC pregnancy test and it is positive. Which hormone is responsible for producing the positive result?

a. HCG

The nurse is caring for a newborn who is 18 inches long, weighs 4 pounds, 14 ounces, has a head circumference of 13 inches and a chest circumference of 10 inches. Based on these physical findings, assessment for which condition has the highest priority? a. Hyperthermia b. polycythemia c. hyperbilirubinemia d. hypoglycemia

a. Hyperthermia

A new mother who is breastfeeding her 4-week old infant and has type 1 diabetes, reports that her insulins needs have decreased since the birth of her child. What action should the nurse implement? a. Inform her that a decrease for insulin occurs while breastfeeding b. Advice the client to breastfeed more frequently c. Counsel her to increase her calories retake d. Schedule an appointment for the client with diabetic nurse educator

a. Inform her that a decrease for insulin occurs while breastfeeding

At 0600 while admitting a woman for a scheduled repeat Caesarean section (C-section), the client tells the nurse that she drank a cup of coffee at 0400 because she wanted to avoid getting a headache. Which action should the nurse take first?

a. Inform the anesthesia care provider

Following the vaginal delivery of a large-for-gestation-age (LGA) infant a woman is admitted to the intensive care unit due to postpartum hemorrhaging. The client's medical record lists have client's religion as Jehovah's Witness. What action should the nurse take?

a. Inform the client of the critical need for a blood transfusion

A term multigravida who is receiving oxytocin (Pitocin) for labor augmentation, is requesting pain medication. Review of the client's record indicates that she was medicated 30 minutes ago with butorphanol (Stadol) 2 mg and promethazine (Phenergan) 25 mg IV push. Vaginal exam reveals that the clients cervical dilation is 3 cm, 70% effaced, and at a 0 station. What action should the nurse implement?

a. Instruct the client to use deep breathing during a contraction

Which nursing intervention is most helpful in relieving postpartum uterine contractions or "afterpains?" a. Lying prone with a pillow on the abdomen b. Using a breast pump c. Massaging the abdomen d. Giving oxytocic medications

a. Lying prone with a pillow on the abdomen Lying prone (A) keeps the fundus contracted and is especially useful with multiparas, who commonly experience afterpains due to lack of uterine tone.

When explaining "postpartum blues" to a client who is 1 day postpartum, which symptoms should the nurse include in the teaching plan? (Select all that apply.) a. Mood swings b. Panic attacks c. Tearfulness d. Decreased need for sleep e. Disinterest in the infant

a. Mood swings c. Tearfulness "Postpartum blues" is a common emotional response related to the rapid decrease in placental hormones after delivery and include mood swings (A), tearfulness (C), feeling low, emotional, and fatigued.

During the admission of a newborn, the nurse identifies a localized swelling that does not cross the suture line on the posterior area of the parietal bone. What action should the nurse implement?

a. Notify the pediatrician of the cephalhematoma

A diabetic client delivers a full-term large for gestation- age (LGA) infant who is jittery action should the nurse take first?

a. Obtain a blood glucose level

Following the vaginal delivery of a 10-pound infant, the nurse assesses a new mothers vaginal bleeding and finds that she has saturated two pads in 30minutes and has a boggy uterus. What action should the nurse implement first?

a. Perform fundal massage until firm

The parents of a male newborn have signed an informed consent for circumcision. which intervention should the nurse implement upon completion of the circumcision? a. Place petroleum gauze dressings on the site b. wrap the infant in warm receiving blankets c. Give a PRN dose of liquid acetaminophen d. Offer a pacifier dipped in glucose water

a. Place petroleum gauze dressings on the site

The nurse's assessment of a preterm infant reveals decreased muscle tone, signs of respiratory difficulty, irritability, and mottled, cool skin. Which intervention should the nurse implement first? a. Position radiant warmer over the crib b. Assess the infants blood glucose level c. Nipple feed 1 ounce 1% glucose in water d. Place the infant in side-lying position

a. Position radiant warmer over the crib

A primigravida at 36 weeks gestation, who is Rh negative, experienced abdominal trauma in a motor vehicle collision. Which assessment finding is more important for the nurse to report to the HCP?

a. Positive fetal hemoglobin testing

The nurse is receiving a report for a laboring client who arrived in the ER with ruptured membranes that the client did not recognize. What is the priority nursing action to implement when the client is admitted to the labor and delivery suite. a. Prepare to start at IV * b. take the clients temp c. begin a pad count d. monitor amniotic fluid for meconium

a. Prepare to start at IV *

A client receiving epidural anesthesia begins to experience nausea and becomes pale and clammy. What intervention should the nurse implement first? a. Raise the foot of the bed b. Assess for vaginal bleeding c. Evaluate the fetal heart rate d. Take the client's blood pressure

a. Raise the foot of the bed These symptoms are suggestive of hypotension which is a side effect of epidural anesthesia. Raising the foot of the bed (A) will increase venous return and provide blood to the vital areas. Increasing the IV fluid rate using a balanced non-dextrose solution and ensuring that the silent is in a lateral position are also appropriate interventions.

The nurse is providing discharge teaching for a client who is 24 hours postpartum. The nurse explains to the client that her vaginal discharge will change from red to pink and then to white. The client asks, "What if I start having red bleeding after it changes?" What should the nurse instruct the client to do? a. Reduce activity level and notify the healthcare provider b. Go to bed and assume a knee-chest position c. Massage the uterus and go to the emergency room d. Do not worry as this is a normal occurrence

a. Reduce activity level and notify the healthcare provider Lochia should progress in stages from rubra (red) to serosa (pinkish) to alba (whitish), and not return to red. The return to rubra usually indicates subinvolution of infection.

A primigravida client confides in the nurse that her sister told her that she should eliminate all salt once she is at 26 weeks' gestation because it will eliminate fluid retention and swelling. How should the nurse respond? a. Salt foods lightly during cooking but add no additional salt at the table. b. eliminate all added salt from the diet to improve kidney function during pregnancy c. limit grain, meat and milk products which are significant sources of sodium d. use canned food products to obtain salt because it is easier to monitor salt intake

a. Salt foods lightly during cooking but add no additional salt at the table

pregnant, homeless woman who has received no prenatal care presents to the clinic in her third trimester because she is having vaginal bleeding but reports that she is not in pain. Ultrasound reveals a placenta previa. Which actions should the nurse implement?

a. Schedule weekly prenatal appointments

Which content should the nurse plan to include in a nutrition class for pregnant adolescents? (select all that apply) a. Take iron and calcium supplements daily b. Gain no more than 15 pounds during the pregnancy c. Increase food intake by 300 to 400 calorie /day d. Take folic acid supplement daily e. Maintain current protein intake

a. Take iron and calcium supplements daily c. Increase food intake by 300 to 400 calorie /day d. Take folic acid supplement daily e. Maintain current protein intake

A new mother asks the nurse about an area of swelling on her baby head near the posterior fontanel that lies across the suture lines. How should the nurse respond? a. That's called caput succedaneum. It will absorb and cause no problems. b. That is called a cephalhematoma. It will cause no problems. c. That is called a cephalhematoma. It can cause jaundice as it is. d. That is called caput succedaneum. It will have to be drained.

a. That's called caput succedaneum. It will absorb and cause no problems.

A client at 20 weeks gestation comes to antepartal clinic complaining of vaginal warts (human papilloma virus HPV). What information should the nurse provide this client?

a. This client should be treated with acyclovir (Zovirax)

A 42-week gestational client is receiving an intravenous infusion of oxytocin (Pitocin) to augment early labor. The nurse should discontinue the oxytocin infusion for which pattern of contractions? a. Transition labor with contractions every 2 minutes, lasting 90 seconds each a. Early labor with contractions every 5 minutes, lasting 40 seconds each c. Active labor with contractions every 31 minutes, lasting 60 seconds each d. Active labor with contractions every 2 to 3 minutes, lasting 70 to 80 seconds each

a. Transition labor with contractions every 2 minutes, lasting 90 seconds each Contractions pattern (A) describes hyperstimulation and an inadequate resting time between contractions to allow for placental perfusion. The oxytocin infusion should be discontinued.

A client who delivered a healthy newborn an hour ago asks the nurse when she can go home. Which information is most important for the nurse to provide the client?

a. When there is no significant vaginal bleeding

The nurse is discussing involution with a postpartum client. Which statement best indicates that the client understands the effect of breastfeeding on the resumption of menstrual cycle?

a. While i am breastfeeding, my period may be delayed

One day after vaginal delivery of a full-term baby, a postpartum client's white blood cell count is 15,000/mm3. What action should the nurse take first? a. check the differential, since the WBC is normal for this client * b. notify the HCP, since this finding is indicative of infection c. assess the client's temperature, pulse and respirations q4h d. assess the clients perineal area for signs of perineal hematoma

a. check the differential, since the WBC is normal for this client *

The nurse is caring for a woman with a previously diagnosed heart disease who is in the second stage of labor. Which assessment findings are of greatest concern? a. edema, basilar rales, and an irregular pulse b. Increased urinary output, and tachycardia c. Shortness of breath, bradycardia, and hypertension d. Regular heart rate, and hypertension

a. edema, basilar rales, and an irregular pulse Edema, basilar rales, and an irregular pulse (A) indicate cardiac decompensation and require immediate intervention.

A client at 30 weeks gestation is being treated in the emergency department for a broken finger. The nurse assesses the FHR while the client is in a sitting position and has a heart rate of 92 beats per minute. What intervention is most important for the nurse to perform? a. encourage the client to empty her bladder * b. determine the maternal pulse rate c. instruct the client to drink a glass a juice d. place the client in a supine position

a. encourage the client to empty her bladder *

A full-term infant is admitted to the newborn nursery 2 hours after delivery. The delivery record indicates that the mother is positive for HIV and received zidovudine AZT IV during labor. What action should the nurse implement? a. ensure that AZT is given within 6 hours after birth b. assess for the presence of the Moro reflex c. collect venous specimen for serum glucose level d. obtain consent for the Hep B vaccine

a. ensure that AZT is given within 6 hours after birth

The nurse is teaching a client with gestational diabetes about nutrition and insulin need for pregnancy. Which content should the nurse include in this client teaching plan?

a. increase daily caloric intake is needed

A one-day-old neonate develops a cephalohematoma. The nurse should closely assess this neonate for which common complication? a. jaundice* b. brain damage c. poor appetite d. hypoglycemia

a. jaundice*

When performing daily head to toe assessment of a 1-day old newborn the nurse observes yellow tint to the skin on the forehead, sternum and abdomen. What action should the nurse take? a. measure bilirubin levels using transcutaneous bilirubinometer. b. review maternal medical records for blood type and Rh factor c. Prepare the newborn for phototherapy d. Evaluate cord Coombs test results

a. measure bilirubin levels using transcutaneous bilirubinometer.

A client at 34 weeks gestation comes to the birthing center complaining of vaginal bleeding that began one hour ago. The nurse's assessment reveals approximately 30 ml of bright red vaginal bleeding, FHR of 130 to 140 beats/min, no contraction, and no complaints of pain. What is the most likely case of this client's bleeding? a. placenta previa b. a ruptured blood vessel in the vaginal vault c. normal bloody show indicating initiation of labor d. abruptio placenta

a. placenta previa

Which type of anesthesia, used with a client in labor, produces a loss of sensation only to the vagina and perineum? a. pudendal block b. epidural block c. saddle block d. paracervical block

a. pudendal block

A new mother asks the nurse about an area of swelling on her baby's head near the posterior fontanel that lies across the suture line. How should the nurse respond? a. that is called a caput succedaneum. it will absorb and cause no problems* b. that is called a cephalohematoma. it can cause jaundice as it is absorbed c. that is called a cephalohematoma. it will cause no problems d. that is called a caput succedaneum. it will have to be drained

a. that is called a caput succedaneum. it will absorb and cause no problems*

Clients who are HIV positive are encouraged to bottle-feed their infants because:

a. the HIV virus is transmitted through breastmilk.

When do the anterior and posterior fontanels close?

anterior fontanel closes at 12 to 18 months and the posterior by the end of the second month.

In determining the one-minute Apgar score of a male infant, the nurse assesses a heart rate of 120 beats per minute and 44 respirations per minute. He has a loud cry with stimulation, good muscle tone and his color is acrocyanotic. What Apgar score should the nurse assign? a. 7 b. 9 ** c. 10 d. 8

b. 9 **

A woman who delivered a 9-pound baby boy by cesarean section under spinal anesthesia is recovering in the post anesthesia care unit. Her fundus is firm at the umbilicus and a continues to trickle bright red blood with no clots from the vagina in observed by the nurse. Which actions should the nurse implemented. a. Massage the fundus b. Assess her blood pressure c. Apply ice pack to perineum d. Let the infant breast feed

b. Assess her blood pressure

The nurse is planning care for a client at 30 weeks gestation who is experiencing preterm labor. What maternal prescription is most important in preventing this fetus from developing respiratory distress syndrome? a. terbutaline (Brethine) 0.25 mg SubQ Q15 mins x 3 b. Betamethasone (Celestone) 12 mg deep IM c. Butorphanol 1 mg IV push q2h PRN pain d. Ampicillin 1-gram IV push q8h

b. Betamethasone (Celestone) 12 mg deep IM

26-week gestational primigravida who is carrying twins is seen in the clinic today. Her fundal height in measured at 29cm. Based on these findings what actions the nurse implement. a. Notify the healthcare provider of the finding b. Document the finding in the medical record c. Schedule the client for a biophysical profile d. Request another nurse measure the fundus

b. Document the finding in the medical record

The total bilirubin level of a 36-hour, breastfeeding newborns is 14 mg/dl. Based on this finding, which intervention should the nurse implement? a. Feed the newborn sterile water hourly b. Encourage the mother to breastfeed frequently c. Assess the newborn's blood glucose level d. Encourage the mother to breastfeed frequently

b. Encourage the mother to breastfeed frequently The normal total bilirubin level is 6 to 12 mg/dl after Day 1 of life. This infant's bilirubin is beginning to climb, and the infant should be monitored to prevent further complications. Breast milk provides calories and enhances GI motility, which will assist the bowel in eliminating bilirubin (C).

A client in active labor complains of cramps in her leg. What intervention should the nurse implement? a. Ask the client if she takes a daily calcium tablet b. Extend the leg and dorsiflex the foot c. Lower the leg off the side of the bed d. Elevate the leg above the heart b. Extend the leg and dorsiflex the foot

b. Extend the leg and dorsiflex the foot Dorsiflexing the foot by pushing the sole of the foot forward or by standing (if the client is capable) (B) and putting the heel of the foot on the floor is the best means of relieving leg cramps.

Which maternal behavior is the nurse most likely to see when a new mother receives her infant for the first time? a. She eagerly reaches for the infant, undresses the infants, and examines the infant completely b. Her arms and hands receive the infant and she then traces the infant's profile with her fingertips c. Her arms and hands receive the infant and she then cuddles the infant to her own body d. She eagerly reaches for the infant and then holds the infant close to her own body

b. Her arms and hands receive the infant and she then traces the infant's profile with her fingertips Attachment/bonding theory indicates that most mothers will demonstrate behaviors described in (B) during the first visit with the newborn, which may be at delivery of later.

The nurse is planning a class for pregnant women in the first trimester of pregnancy. Which information is most important for the nurse to include in the class? a. Plan rest periods and increase sleep time to an hour per day when fatigue b. If any vaginal bleeding occurs, notify the healthcare provider immediately c. Since eating often relieves nausea, carry low fat snacks to eat whenever nausea occurs d. If morning dizziness occurs, rise slowly and sit on the side of the bed for one minute

b. If any vaginal bleeding occurs, notify the healthcare provider immediately

Which action should the nurse take if an infant, who was born yesterday weighing 7.5lbs (3,317grams) weights 7 lbs. (3,175grams) today. a. Monitor the stool and urine output of the neonate for the last 24 hours b. Inform and assure the mother that this is a normal weight loss c. Encourages the mother to increase frequency of breastfeeding. d. After verifying the accuracy of the weight, notify the healthcare provider.

b. Inform and assure the mother that this is a normal weight loss

A client who is attending antepartum classes asks the nurse why her healthcare provider has prescribed iron tablets. The nurse's response is based on what knowledge? a. Supplementary iron is more efficiently utilized during pregnancy b. It difficult to consume 18 mg of additional iron by diet alone c. Iron absorption is decreased in the GI tract during pregnancy d. Iron is needed to prevent megaloblastic anemia in the last trimester

b. It difficult to consume 18 mg of additional iron by diet alone Consuming enough iron-containing foods to facilitate adequate fetal storage of iron and to meet the demands of pregnancy is difficult (B) so iron supplements are often recommended.

The nurse identifies crepitus when examining the chest of a newborn who was delivered vaginally. Which further assessment should the nurse perform? a. Elicit positive scarf sign on the affected side b. Observe for an asymmetrical Moro (startle) reflex c. Watch for swelling of fingers on the affected side d. Note paralysis of affected extremity and muscles

b. Observe for an asymmetrical Moro (startle) reflex The most common neonatal birth trauma due to vaginal delivery is fracture of the clavicle. Although an infant may be asymptomatic, a fracture clavicle should be suspected is an infant has limited use of the affected arm malposition of the arm, an asymmetric Moro reflex (B), crepitus over the clavicle, focal swelling or tenderness, or cries when the arm is moved.

A newborn infant is receiving immunization prior discharge. Which action should the nurse implement? a. Give the first dose of the vaccine for rotavirus if any have diarrhea now. b. Obtain signed consent from the mother for administration of hepatitis B vaccine c. Prepare the first dose for DTaP d. Ask the mother if she wants the infant immunized for

b. Obtain signed consent from the mother for administration of hepatitis B vaccine

A pregnant client mentions in her history that she changes a cat's litter box daily. Which test should the nurse anticipate the HCP to prescribe? a. Biophysical profile b. TORCH screening c. Fern Test d. amniocentesis

b. TORCH screening

A 39-week gestational multigravida is admitted to labor and delivery spontaneous rupture of membranes and contraction occurring 2 to 3 minutes. A vaginal exam indicates that the cervix is dilated 6cm, 90% effaced and the fetus is at a +2 station. During the last 45 minutes the fetal heart rate has ranged between 170 and 180 beats/minute. What action should the nurse implement? a. Obtain a blood specimen for hemoglobin b. Take an oral maternal temperature c. Straight Catheterize client d. Send amniotic fluid for analysis

b. Take an oral maternal temperature

The healthcare provider prescribes 10 units/L of oxytocin (Pitocin) via IV drips to augment a client's labor because she is experiencing a prolonged active phase. Which finding would cause the nurse to immediately discontinue the oxytocin? a. uterus soft b. contraction duration of 100 seconds c. four contractions in 10 minutes d. Early deceleration of fetal heart rate

b. contraction duration of 100 seconds

The nurse is performing a newborn assessment. Which symptom, if present in a newborn would indicate respiratory distress? a. respiratory rate of 50 breaths per min b. flaring of the nares * c. shallow and irregular respirations d. abdominal breathing with synchronous chest movement

b. flaring of the nares *

A primigravida at 40 weeks gestation is contracting q2 minutes and her cervix is 9 cm dilated and 100% effaced. The FHR is 120 beats/minute. The client is screaming, and her husband is alarmed. Which intervention should the nurse implement? a. notify the rapid response team b. have delivery table set up * c. ask the husband to step out d. administer a PRN narcotic

b. have delivery table set up *

A 5-day old infant with a serum bilirubin of 19 mg/dl is being discharged from the hospital. Which instruction should the nurse include in the discharge teaching plan? a. breastfeed infant every 4 hours b. monitor skin and eyes for yellow tinge c. reposition the infant every 2 hours d. change diapers every hour

b. monitor skin and eyes for yellow tinge

Four clients at full term present to the labor and delivery unit at the same time. which client should the nurse assess first? a. primipara with vaginal show and leaking membranes b. primipara with burning on urination and urinary frequency c. multipara scheduled for a non-stress test and biophysical profile d. multipara with contractions occurring every 3 minutes

b. primipara with burning on urination and urinary frequency

The nurse is caring for a client following an emergency cesarean delivery under a general anesthesia. Which assessment finding, occurring in the first 8 hours after delivery, is more critical and requires immediate intervention? a. mild nausea and anorexia b. uterine atony * c. a positive Homan's sign d. Respiratory rate 12

b. uterine atony *

A 33-year-old client at 9 weeks gestation tells the nurse that while she has "cut down," she still has at least one alcoholic drink every evening before bedtime. What intervention should the nurse implement? a) Notify child protective services of the client's illicit drug use and probable child endangerment b) Praise the client for her actions and offer to discuss ways to decrease consumption even more c) Insist that the client stop all alcohol use and draw a blood alcohol level at each prenatal visit

c) Insist that the client stop all alcohol use and draw a blood alcohol level at each prenatal visit

When assessing a pregnant woman at 39 weeks gestation who is admitted to labor and delivery, which finding is most important to report to the HCP? a. +1 pedal edema b. 130/70 blood pressure c. 101.2 F oral temp * d. +1 proteinuria

c. 101.2 F oral temp *

Vaginal examination reveals that a laboring clients' cervix is dilated to 2 cm, 70% effaced, with the presenting part at -2 stations. The client tells the nurse, "I need my epidural now! This hurt!" the nurses' response to the client should be based on what information? a. the client should be dilated to at least 8 cm before receiving an epidural b. the baby needs to be at a zero station before an epidural can be administered c. Administering an epidural at this point would slow the labor process * d. the client will need to be catheterized before the epidural can be administered.

c. Administering an epidural at this point would slow the labor process *

A mother who is breastfeeding her baby receives instructions from the nurse. Which instruction is most effective to prevent nipple soreness? a. Wear a cotton bra b. Increase nursing time gradually c. Correctly place the infant on the breast d. Manually express a small amount of milk before nursing

c. Correctly place the infant on the breast The most common cause of nipple soreness is incorrect positioning (C) of the infant on the breast, e.g., grasping too little of the areola or grasping on the nipple.

The nurse is teaching care of the newborn to a group of prospective parents and describes the need for administering antibiotic ointment into the eyes of the newborn. Which infectious organism will this treatment prevent from harming the infant? a. Herpes b. Staphylococcus c. Gonorrhea d. Syphilis

c. Gonorrhea Erythromycin ointment is instilled into the lower conjunctive of each eye within 2 hours after birth to prevent ophthalmic neonatorum, an infection caused by gonorrhea, and inclusion conjunctivitis, an infection caused by chlamydia (C). The infant may be exposed to these bacteria when passing the birth canal.

The nurse is caring for a postpartum client who is complaining of severe pain and a feeling of pressure in her perineum. Her fundus is firm, and she has a moderate lochia flow. On inspection the nurse finds that a perineal hematoma is beginning to form. Which assessment findings show the nurse obtain first? a. Abdominal contour and bowel sounds b. Hemoglobin and hematocrit c. Heart rate and blood pressure d. Urinary output and IV fluid intake

c. Heart rate and blood pressure

At 0600 while admitting a woman for a scheduled repeat c section, the client tells the nurse that she drank a cup of coffee at 0400 because she wanted to avoid getting a headache. What action should the nurse take first? a. Ensure preoperative lab results are available b. Start prescribed IV with Lactated Ringers c. Inform the anesthesia care provider d. Contact the client's obstetrician

c. Inform the anesthesia care provider

A 33-year-old client at 9 weeks gestation tells the nurse that while she has "cut down," she still has at least one alcoholic drink every evening before bedtime. What intervention should the nurse implement? a. Notify child protective services of the client's illicit drug use and probable child endangerment b. Praise the client for her actions and offer to discuss ways to decrease consumption even more c. Insist that the client stop all alcohol use and draw a blood alcohol level at each prenatal visit

c. Insist that the client stop all alcohol use and draw a blood alcohol level at each prenatal visit

woman in her third trimester of pregnancy has been in active labor for the past 8 hours and cervix dialed 3cm. The nurse's assessment findings and electronic fetal monitoring (EFM) are consistent with hypotonic dystocia, and the healthcare provider prescribes an oxytocin drip. Which data is most important for the nurse to monitor? a. Clients hourly blood pressure b. Preparation for emergency cesarean birth c. Intensity, interval, and length of contractions d. Checking the perineum for bulging

c. Intensity, interval, and length of contractions

The nurse is counseling a woman who wants to become pregnant. The woman tells the nurse that she has a 36-day menstrual cycle and the first day of her menstrual period was January *. The nurse correctly calculates that the woman's next fertile period is a. January 14-15 b. January 22-23 c. January 30-31 d. February 6-7

c. January 30-31 This woman can expect her next period to begin 36 days from the first day of her last menstrual period - the cycle begins at the first day of the cycle and continues to the first day of the next cycle. Her next period would, therefore, begin on February 13. Ovulation occurs 14 days before the first day of the menstrual period. Therefore, ovulation for this woman would occur January 31 (C).

The nurse assesses a client admitted to the labor and delivery unit and obtains the following data: dark red vaginal bleeding, uterus slightly tense between contractions, BP 110/68, FHR 110 beats/minute, cervix 1 cm dilated and uneffaced. Based on these assessment findings, what intervention should the nurse implement? a. Insert an internal fetal monitor b. Assess for cervical changes q1h c. Monitor bleeding from IV sites d. Perform Leopold's maneuvers

c. Monitor bleeding from IV sites Monitoring bleeding from peripheral sites (C) is the priority intervention. This client is presenting with signs of placental abruption. Disseminated intravascular coagulation (DIC) is a complication of placental abruption, characterized by abnormal bleeding.

A client at 30-weeks gestation, complaining of pressure over the pubic area, is admitted for observation. She is contracting irregularly and demonstrates underlying uterine irritability. Vaginal examination reveals that her cervix is closed, thick, and high. Based on these data, which intervention should the nurse implement first? a. Provide oral hydration b. Have a complete blood count (CBC) drawn c. Obtain a specimen for urine analysis d. Place the client on strict bedrest

c. Obtain a specimen for urine analysis Obtaining a urine analysis (C) should be done first because preterm clients with uterine irritability and contractions are often suffering from a urinary tract infection, and this should be ruled out first.

A client is admitted with the diagnosis of total placenta previa. Which finding is most important for the nurse to report to the healthcare provider immediately? a. Heart rate of 100 beats/minute b. Variable fetal heart rate c. Onset of uterine contractions d. Burning urination

c. Onset of uterine contractions Total (complete) placenta previa involves the placenta covering the entire cervical is (opening). The onset of uterine contractions (C) places the client at risk for dilation and placental separation, which causes painless hemorrhaging.

One hour after delivery the nurse is unable to palpate the uterine funds of a client who had an epidural and notes a large amount of lochia on the perineal pad. The nurse massages at the umbilicus and obtains current vital signs. Which intervention should the nurse implement next. a. Document number of pad changes in the last hour b. Provide bedpans to void if unable to ambulate c. Palpate the supra cubic area for bladder distention d. Increases the rate of the oxytocin infusion

c. Palpate the supra cubic area for bladder distention

A newborn, whose mother is HIV positive, is scheduled for follow-up assessments. The nurse knows that the most likely presenting symptom for a pediatric client with AIDS is: a. shortness of breath b. joint pain c. a persistent cold d. organomegaly

c. a persistent cold Respiratory tract infections commonly occur in the pediatric population. However, the child with AIDS has a decreased ability to defend the body against these infections and often the presenting symptom of a child with AIDS is a persistent cold (C).

A client at 40-weeks' gestation presents to the obstetrical floor and indicates that the amniotic membranes ruptured spontaneously at home. She is in active labor and feels the need to bear down and push. What information is most important for the nurse to obtain first? a. the estimated amount of fluid b. time the membranes ruptured c. color and consistency of the fluid d. any odor noted when membranes ruptured.

c. color and consistency of the fluid

The nurse is conducting a postpartum teaching with a mother who is breastfeeding her infant. When discussing birth control, which method should the nurse recommend to this client as best for her to use in preventing an unwanted pregnancy? a. combined estrogen- progesterone oral contraceptives b. breastfeed exclusively at least every 3 to 4 hours c. condoms and contraceptive foam or gel d. rhythm method (natural family planning)

c. condoms and contraceptive foam or gel

A 26-week gestation primigravida who is carrying twins is seen in the clinic today. Her final height is measured at 29 cm. Based on these findings, what action should the nurse implement? a. notify the HCP of the finding b. schedule the client for a biophysical profile c. document the finding in the medical record d. request another nurse measure the fundus

c. document the finding in the medical record

The current vital signs for a primipara who delivered vaginally during the previous shift are: temperature 100.4 F, heart rate 58 beats/min, respiratory rate 16 breaths/min, and blood pressure 130/74. What action should the nurse implement? a. administer a PRN dose of acetaminophen b. report heart rate to HCP c. document the vital signs in the record d. assess the perineum for excessive lochia

c. document the vital signs in the record

A client at 30 weeks gestation reports that she has not felt the baby move in the last 24 hours. Concerned, she arrives in a panic at the obstetric clinic where she is immediately sent to the hospital. Which assessment finding warrants immediate intervention by the nurse? a. the onset of uterine contractions b. leaking amniotic fluid c. fetal heart rate 60 beats/min* d. ruptured amniotic membrane

c. fetal heart rate 60 beats/min*

A client is admitted to the postpartum unit and tells the nurse she had rheumatic fever as a child, which resulted in some "heart damage." The nurse knows that this client is at particular risk for developing heart failure during the immediate postpartum period. Based on this client's history, which nursing diagnosis has the highest priority? a. sleep deprivation b. risk for infection c. fluid volume excess * d. nausea and vomiting

c. fluid volume excess *

The nurse receives a newborn within the first minutes after a vaginal delivery and intervenes to establish adequate respirations. What priority should the nurse address to ensure the newborn's survival? a. fluid balance b. hypoglycemia c. heat loss d. bleeding tendencies

c. heat loss

A client at 34 weeks gestation is scheduled to travel for business using a commercial airline. Which instruction is most important for the nurse to provide this client? a. explore the availability of medical care at the destination site b. request an aisle seat in a row that is not designated as an exit row * c. perform ankle flexion and extension several times throughout the trip d. wear non-constricting clothing to prevent edema of the feet and hands

c. perform ankle flexion and extension several times throughout the trip Pregnancy is a stage of hypercoagulation. The clotting factors in the body will increase and the platelet count also will increase, especially towards the end of the pregnancy. Along with this the gravid uterus will compress the blood vessels of the lower extremities and this may leads to sluggishness of venous return. This slow venous return leads to the pooling of blood in the lower extremities

The nurse should encourage the laboring client to begin pushing when a. there is only an anterior or posterior lip of cervix left b. the client describes the need to have a bowel movement c. the cervix is completely dilated d. the cervix is completely effaced

c. the cervix is completely dilated Pushing begins with the second stage of labor, i.e., when the cervix is completely dilated (A, B, and D), the cervix can become edematous and may never completely dilate, necessitating an operative delivery. Many primigravida's begin active labor 100% effaced and then proceed to dilate.

A woman who is trying to get pregnant tells the nurse that she was very disappointed several months ago when she was informed that her positive pregnancy test was a false positive. Which method of determining pregnancy provides the greatest degree of accuracy? a. complaints of feeling tired all the time b. presence of amenorrhea for 2 months c. visualization of implantation by vaginal ultrasound d. maternal blood serum tests positive for alpha-fetoprotein

c. visualization of implantation by vaginal ultrasound

A client who is 3 weeks postpartum tells the nurse. "I am so tired all the time. I didn't know having a baby would be so hard." What response should the nurse provide. a) It is common to feel exhausted for the first 3 months. Try to sleep when the baby sleeps. b) It is normal to feel tired for the first couple weeks. Be patient with yourself and rest more. c) You should not be doing any housework. Are any of your family members helping you? d) Adjusting to a new baby can be difficult. Tell me more about any help you are receiving.

d) Adjusting to a new baby can be difficult. Tell me more about any help you are receiving.

When assessing a pregnant woman AT 39-weeks' gestation who is admitted to labor and delivery which finding is most important to report to the health care provider? a. + proteinuria b. 130/70 blood pressure c. + pedal edema d. 101.2 oral temperature

d. 101.2 oral temperature

A pregnant woman comes to the prenatal clinic for an initial visit. In reviewing her childbearing history, the client indicated that she has delivered premature twins, one full- term baby, and has had no abortions. Which GTPAL should the nurse document in this client's record? a. 3-1-2-0-3 b. 4-1-2-0-3 c. 2-1-2-1-2 d. 3-1-1-0-3

d. 3-1-1-0-3 (D) describes the correct GTPAL. The client has been pregnant 3 times including the current pregnancy (G-3). She had one full-term infant (T-1). She also had a preterm (P- 1) twin pregnancy (a multifetal gestation is considered one birth when calculating parity). There were no abortions (A-0), so this client has a total of 3 living children.

A client who is 3 weeks postpartum tells the nurse. "I am so tired all the time. I didn't know having a baby would be so hard." What response should the nurse provide. a. It is common to feel exhausted for the first 3 months. Try to sleep when the baby sleeps. b. It is normal to feel tired for the first couple weeks. Be patient with yourself and rest more. c. You should not be doing any housework. Are any of your family members helping you? d. Adjusting to a new baby can be difficult. Tell me more about any help you are receiving

d. Adjusting to a new baby can be difficult. Tell me more about any help you are receiving

The nurse is caring for a laboring client who is GBS+ (Group B streptococcus). Which immediate treatment is indicated for this client? a. Administration of Pitocin b. Artificial rupture of the membrane c. Amnioinfusion for the baby d. Administration of antibodies

d. Administration of antibodies

One hour after giving birth to an 8-pound infant, a client's lochia rubra has increased from small to large and her fundus is boggy despite massage. The client's pulse is 84 beats/minute and blood pressure is 156/96. The healthcare provider prescribes Methergine 0.2 mg IM x 1. What action should the nurse take immediately? a. Give the medication as prescribed and monitor for efficacy b. Encourage the client to breastfeed rather than bottle feed c. Have the client empty her bladder and massage the fundus d. Call the healthcare provider to question the prescription

d. Call the healthcare provider to question the prescription Methergine is contraindicated for clients with elevated blood pressure, so the nurse should contact the healthcare provider and question the prescription (D).

Which assessment finding should the nursery nurse report to the pediatric healthcare provider? a. Blood glucose level of 45 mg/dl b. Blood pressure of 82/45 mmHg c. Non-bulging anterior fontanel d. Central cyanosis when crying

d. Central cyanosis when crying An infant who demonstrates central cyanosis when crying (D) is manifesting poor adaptation to extrauterine life which should be reported to the healthcare provider for determination of a possible underlying cardiovascular problem.

pregnant woman who is at 10-weeks' gestation and is 35 years of age tells the nurse that she is concerned about the possibility of having a baby with Down Syndrome. Which information should the nurse provide this client? a. an amniocentesis conducted at 24 weeks' gestation confirms or denies Down Syndrome in the fetus b. maternal serum Human Chorionic Gonadotropic (HCG) can identify Down Syndrome at 6 weeks of gestation c. Weekly fundal height measurements are a noninvasive method used to check for Down Syndrome d. Chorionic villus sampling at 12 weeks gestation is the earliest screening test used to identify Down Syndrome *

d. Chorionic villus sampling at 12 weeks gestation is the earliest screening test used to identify Down Syndrome *

The nurse examines a client who is admitted in active labor and determines the cervix is 3cm dilated 50% effaced, and the presenting part is at 0 stations. An hour later, she tells the nurse that she wants to go to the bathroom. Which action should the nurse implement first. a. Check the pH of the vaginal fluid b. Review the fetal heart rate pattern c. Palpate the client's bladder d. Determine cervical dilation

d. Determine cervical dilation

A client at 32-weeks gestation comes to the prenatal clinic with complaints of pedal edema, dyspnea, fatigue, and a moist cough. Which question is most important for the nurse to ask this client? a. Which symptom did you experience first? b. Are you eating large amounts of salty foods? c. Have you visited a foreign country recently? d. Do you have a history of rheumatic fever?

d. Do you have a history of rheumatic fever? Clients with a history of rheumatic fever (D) may develop mitral valve prolapse, which increases the risk for cardiac decompensation due to the increased blood volume that occurs during pregnancy, so obtaining information about the client's health history is priority.

At 6 weeks gestation the rubella titer of a client medication indicates she is non- immune. When is the best time to administer a rubella vaccine to this client? a. After the client stops breastfeeding b. Immediately, at 6-weeks' gestation to protect fetus c. After the client reaches 20-weeks gestations d. Early postpartum within 72 hours of delivery

d. Early postpartum within 72 hours of delivery

A term multigravida, who is receiving oxytocin for labor augmentation is requesting pain medication. Review of the clients record indication that she was medicated 30minutes ago with butorphanol (Stadol) 2mg and promethazine (Phenergan) 25mg IV push. Vaginal examination reveals that the client cervical dilation is 3cm, 70% effaced, and at a 0 station. What action should the nurse implement? a. Discontinue the Pitocin infusion b. Medicate the client with an additional 1mg of Stadol IV push c. Notify the healthcare provider d. Instruct the client to use deep breathing during contraction

d. Instruct the client to use deep breathing during contraction

A client who is in the second trimester of pregnancy tells the nurse that she wants to use herbal therapy. Which response is best for the nurse to provide? a. Herbs are a corner stone of good health to include in your treatment b. Touch is also therapeutic in relieving discomfort and anxiety c. Your healthcare provider should direct treatment options for herbal therapy d. It is important that you want to take part in your care

d. It is important that you want to take part in your care The emphasis of alternative and complementary therapies, such as herbal therapy, is that the client is viewed as a whole being, capable of decision-making and an integral part of the health care team, so (D) recognizes the client's request.

A client who gave birth to a healthy 8-pound infant 3 hours ago is admitted to the postpartum unit. Which nursing plan is best in assisting this mother to bond with her newborn infant? a. Encourage the mother to provide total care for her infant b. Provide privacy, so the mother can develop a relationship with the infant c. Encourage the father to provide most of the infant's care during hospitalization d. Meet the mother's physical needs and demonstrate warmth toward the infant

d. Meet the mother's physical needs and demonstrate warmth toward the infant It is most important to meet the mother's requirement for attention to her needs so that she can begin infant care-taking (D). Nurse theorist Reva Rubin describes the initial postpartum period as the "taking-in phase," which is characterized by maternal reliance on others to satisfy the needs for comfort, rest, nourishment, and closeness to families and the newborn.

An obviously pregnant woman walks into the hospital's emergency department entrance shouting. "Help me! Help me! My baby is coming! Im so afraid!" The nurse determines if delivery is indeed imminent, what action is most important for the nurse to take? a. Determines the gestational age of fetus b. Assess the amount and color of the amniotic fluid c. Obtain peripheral IV access and begin administration of IV fluids d. Provide clear concise instructions in a calm, deliberate manner

d. Provide clear concise instructions in a calm, deliberate manner

The nurse is calculating the estimated date of confinement (EDC) using Nagele's rule for a client whose last menstrual period started on December 1. Which date is most accurate? a. August 1 b. August 10 c. September 3 d. September 8

d. September 8 Calculation of a client's EDC provides baseline data to monitor fetal gestation. Nagele's rule uses the formula: subtract 3 months and add 7 days to the first day of the last normal menstrual period, so December 1 minus 3 months + 7 days is September 8 (D).

After delivery of a normal infant, the mother tells the nurses that she would like to use oral contraceptive. Which finding in the client's health history is a contraindication of the use of contraceptives? a. Previously used intrauterine device (IUD) b. Reported history of stroke within family c. Diagnosed with diabetes mellitus 2 years ago d. Smoked cigarettes prior to becoming pregnant

d. Smoked cigarettes prior to becoming pregnant

A woman who had a miscarriage 6 months ago becomes pregnant. Which instruction is most important is most important for the nurse to provide this client? a. Elevate lower legs while resting b. Increase caloric intake by 200 to 300 calories per day c. Increase water intake to 8 full glasses per day d. Take prescribed multivitamin and mineral supplements

d. Take prescribed multivitamin and mineral supplements A client who has had a spontaneous abortion or still birth in the last 1.5 years should take multivitamin and mineral supplements (D) and maintain a balanced diet because the previous pregnancy may have left her nutritionally depleted.

A 35-year-old primigravida client with severe preeclampsia is receiving magnesium sulfate via continuous IV infusion. Which assessment data indicates to the nurse that the client is experiencing magnesium sulfate toxicity? a. Deep tendon reflexes 2+ b. Blood pressure 140/90 c. Respiratory rate 18/minute d. Urine output 90 ml/4 hours

d. Urine output 90 ml/4 hours Urine outputs of less than 100 ml/4 hours (D), absent DTRs, and a respiratory rate of less than 12 breaths/minute are cardinal signs of magnesium sulfate toxicity

A client at 38 weeks gestation presents to the labor and delivery unit in active labor. Based on which assessment finding should the nurse notify the surgery team to prepare for a primary cesarean section? a. treated ten days ago for Chlamydia b. Group Beta Strep positive c. Positive western blot for HIV d. active herpes lesions on the perineum

d. active herpes lesions on the perineum

The HCP prescribes 10 units/L of oxytocin (Pitocin) via IV drip to augment a client's labor because she is experiencing a prolonged active phase. Which finding would cause the nurse to immediately discontinue the oxytocin? a. early decelerations of FHR b. uterus is soft c. Four contractions in 10 minutes d. contraction duration of 100 seconds *

d. contraction duration of 100 seconds *

A 25-year-old client who had a severe postpartum hemorrhage following the vaginal birth of twins is transferred to the postpartum unit. The nurse knows that assessment for what complication has the highest priority for this client? a. postpartum psychosis b. hard, painful uterine afterpains c. placenta accreta d. disseminated intravascular coagulation*

d. disseminated intravascular coagulation*

The nurse is caring for a client whose fetus died in utero at 32 weeks gestation. After the fetus is delivered vaginally, the nurse implements routine demise protocol and identification procedures. What action is most important for the nurse to take? a. Explain reasons consent for an infant autopsy is needed b. create a memory box of a baby's footprints and photographs c. determine if the mother desires a visit from the clergy d. encourage the mother to hold and spend time with her baby *

d. encourage the mother to hold and spend time with her baby *

A client who is anovulatory and has hyperprolactinemia is being treated for infertility with metformin (Glucophage), menotropins (Repronex, menopur) and HCG. Which side effect should the nurse tell the client to report immediately? a. persistent daytime fatigue b. rapid increase in abdominal girth c. nausea and vomiting d. episodes of headache and irritability

d. episodes of headache and irritability

The mother of a breastfeeding 24 hr old infant is very concerned about the techniques involved in breastfeeding. She calls the nurse with each feeding to seek reassurance that she is "doing it right." She tells the nurse, "I just know my daughter is not getting enough to eat." What response would be best for the nurse to make? a. feed your baby hourly until you feel confident that your child is receiving enough milk b. don't worry, soon your milk will come in, and you will feel how full your breasts are c. since you are so concerned, you should probably supplement breastfeeding with formula d. if your baby's urine is straw-colored, she is getting enough milk*

d. if your baby's urine is straw-colored, she is getting enough milk*

When performing the daily head to toe assessment of a one-day old newborn, the nurse observes a yellow tint to the skin on the forehead, sternum, and abdomen. What action should the nurse take? a. review maternal medical records for blood type and Rh factor b. prepare the newborn for phototherapy c. evaluate cord blood Coombs' test results d. measure bilirubin levels using transcutaneous bilirubinometer

d. measure bilirubin levels using transcutaneous bilirubinometer

A 32-week gestation client has deep tendon reflexes (DTRs) are 4+. What action should the nurse take first? a. assess the urine for proteinuria b. record the finding on a flowsheet c. obtain blood pressure reading d. notify the healthcare provider

d. notify the healthcare provider

Assessment findings of a 3-hour old newborn include: axillary temperature of 97.7 F, heart rate of 140 beats/min with a soft murmur, and irregular respiratory rate at 42 breaths/min. Based on these findings, what action should the nurse implement? a. record findings in electronic medical record b. obtain venous blood sample for glucose level c. attach a pulse oximeter on the heel d. place the infant under the radiant warmer

d. place the infant under the radiant warmer

The nurse examines a client who is admitted in active labor and determines the cervix is 3 cm dilated, 50% effaced, and the presenting part is at 0 station. An hour later, she tells the nurse that she wants to go to the bathroom. Which action should the nurse implement first? a. palpate the client's bladder b. check the pH of the vaginal fluid c. determine cervical dilation d. review the FHR pattern *

d. review the FHR pattern *

postpartum client who is Rh-negative refuses to receive Rho(D) immune globulin (RhoGAM) after delivery of an infant who is Rh-positive. Which information should the nurse provide this client? a. RhoGAM is not necessary unless all her pregnancies are Rh-positive b. RhoGAM prevents maternal antibody formation for future Rh-positive babies c. the mother should receive RhoGAM when the baby is Rh-negative d. the R-positive factor from the fetus threatens her blood cells *

d. the R-positive factor from the fetus threatens her blood cells *

Following a precipitous labor, a postpartum client has a continuous trickling of bright red blood from her vagina. Her uterus is firm, and her vital signs are within normal limits. The nurse determines that this sign may indicate which condition? a. expected course in the fourth stage of labor b. a full urinary bladder c. early postpartum hemorrhage d. the laceration on the cervix *

d. the laceration on the cervix *

A client at 26 weeks gestation recently indicated a yellow discharge from her right breast. How should the nurse respond? a. you need to wear a good support bra b. you need to discuss this with your HCP c. you probably have an infection d. this is normal *

d. this is normal *

The nurse is preparing to draw blood from a newborn to obtain hemoglobin and hematocrit levels. What is the best method to obtain this blood sample? a. use a butterfly, small gauge needle to do a venous puncture on the hand b. draw blood from the infant's antecubital vein using a small gauge needle c. use a small gauge needle to puncture the vastus lateralis d. use a lancet to puncture the outer lateral aspect of the heel *

d. use a lancet to puncture the outer lateral aspect of the heel *

A client who suspects she is pregnant tells the nurse she has a peptic ulcer that is being treated with misoprostol (Cytotec), a synthetic prostaglandin E drug. How should the nurse respond? a. you may have an increased chance of having preeclampsia b. this medication will have no effect on your unborn child c. you may experience postpartum hemorrhage after delivery d. you may be at higher risk for having a spontaneous miscarriage *

d. you may be at higher risk for having a spontaneous miscarriage *

What is megaloblastic anemia caused by?

folic acid deficiency

Define Preeclampsia:

pregnancy complication characterized by high blood pressure and signs of damage to another organ system, often the kidneys. It usually begins after 20 weeks of pregnancy in a woman whose blood pressure had been normal. Characterized by: • Hypertension • Pulmonary edema • Proteinuria • Renal insufficiency • Visual disturbances • Thrombocytopenia • Impaired liver function Use magnesium sulfate for treatment

The nurse should encourage the laboring client to begin pushing when...

the cervix is completely dilated. Pushing begins with the second stage of labor, i.e., when the cervix is completely dilated at 10 cm (C). If pushing begins before the cervix is completely dilated the cervix can become edematous and may never completely dilate, necessitating an operative delivery. Many primigravida's begin active labor 100% effaced and then proceed to dilate.


Kaugnay na mga set ng pag-aaral

Anatomy and Physiology Practice Test

View Set

Accounting Fundamentals - Final Exam Study Questions

View Set

Chapter 1 / Matter, Measurement,​ and Problem Solving

View Set

Ch 3: Cultural and Linguistic Diversity

View Set